Answered You can buy a ready-made answer or pick a professional tutor to order an original one.

QUESTION

MN551 Unit 5 Mid Term Latest 2017/MN551 Unit 5 Mid Term Latest 2017

Question 1.1. A student nurse practitioner asks her preceptor about the origins of different tissues, and their cellular origins during the process of development. Which of the following statements by the preceptor best describes the process of cell differentiation? (Points : 3)“Cells of the hematopoietic system produce the appropriate body cells that are required at each stage of development.”“A single stem cell differentiates into approximately 200 different types of cells.”“A fertilized ovum undergoes a series of divisions, yielding many different cell types.”“Cells differentiate into necessary body cells, peaking after conception, and ceasing near the time of birth.”Question 2.2. A 77-year-old male patient with a diagnosis of stomach cancer has been found to have metastases in his liver. The patient and his family are surprised at this turn of events, stating that they don't see how he could have developed cancer in his liver. Which of the following facts would underlie the reply that the care team provides? (Points : 3)The parenchymal tissue of the liver is particularly susceptible to secondary malignancies.The portal circulatory system brings venous blood from the gastrointestinal tract into the liver.Hepatic stromal tissue shares characteristics with cancerous cells, including lack of anchorage dependence.The proximity of the liver to the stomach allows for direct spread of cancerous cells due to a lack of contact inhibition.Question 3.3. The NP is teaching a group of older adults about the value of including foods containing antioxidants in their diet. Which of the following statements best captures the rationale underlying the NPs advice?(Points : 3)Antioxidants inhibit the actions of reactive oxygen species (ROS).Antioxidants prevent the formation of superoxide dismutase.Antioxidants react nonspecifically with molecules.Antioxidants prevent the occurrence of cell dysplasia. Question 4.4. The nurse practitioner is providing care for a patient with a diagnosis of cirrhosis, and he notes that the patient's sclerae are jaundiced. The nurse practitioner recalls that jaundice is caused by excess accumulation of bilirubin, a pigment that can accumulate in which part of the cell? (Points : 3)NucleusCytoplasmGolgi apparatusRough endoplasmic reticulum (ER) Question 5.5. Which of the following patients of a primary care nurse practitioner would not require extra screening for cancer? (Points : 3)A 51-year-old woman whose grandmother died of breast cancerA 48-year-old man who takes immunosuppressant drugs following a kidney transplantA 50-year-old male who is obese and has a low-fiber, high-fat dietA 38-year-old female with Down syndrome and congenital scoliosisQuestion 6.6. A new older female patient at a long-term care facility has a diagnosis of type 1 neurofibromatosis (NF-1). As part of the intake assessment protocol for the facility, the clinical educator is teaching the care staff about the diagnosis. Which of the following statements most accurately conveys an aspect of neurofibromatosis? (Points : 3)“The neurofibroma lesions are unsightly for the patient, but they are not painful.“Her diagnosis puts her at higher risk of developing a malignant neoplasm.”“She is living with an example of an autosomal recessive disorder.”“The patient is likely to be photosensitive as a result of the disease.”Question 7.7. A child possesses a trait that is the result of the interaction of two different genes, neither of which could have produced the trait independently. Which of the following explanations best captures the genetic explanation for this? (Points : 3)The trait is an expression of multiple alleles.Epistasis has dictated the phenotypic outcome.The phenomenon is an example of polygenic inheritance.The outcome is the result of the interaction between collaborative genes.Question 8.8. Which of the following statements most accurately conveys an aspect of cell injury due to impaired calcium homeostasis? (Points : 3)Normal intracellular calcium ion levels are higher than extracellular levels.Ischemia and certain toxins cause a decrease in cytosolic calcium.Injured cells tend to accumulate calcium.Low calcium levels cause an activation of damaging enzymes.Question 9.9. A group of researchers has identified that the prevalence of two particular genetic disorders share a statistical correlation. Which of the following statements best conveys the genetic rationale for this situation? (Points : 3)There is likely a cause-and-effect relationship between the two genes responsible.The chromosomes containing each gene are likely closely situated.The genes causing each disorder are likely in the same section of the same chromosome.The disorders likely share the same locus. Question 10.10. A male patient of a nurse practitioner has an autosomal dominant disorder. The patient and his partner are considering starting a family. Which of the patient's following statements indicates the patient has an adequate understanding of the genetic basis of this health problem? (Points : 3)“I know there's no way of accurately determining the chance that my child will inherit the disease.”“My children who don't have the disease still run the risk of passing it on to their children.”“I know that new genetic mutations won't occur between generations.”“I know that a single mutant allele is to blame for the health problem.”Question 11.11. As part of an orientation to a genetic counseling practice, a group of medical students is differentiating between autosomal recessive disorders and autosomal dominant disorders. Which of the following statements is true of autosomal recessive disorders? (Points : 3)They can manifest when present in one or both gene pairs.There is a one in two chance of an affected child in each pregnancy with an affected mother.They tend to have a more uniform symptomatology than autosomal dominant disorders.The associated disorders are usually attributable to abnormalities in structural proteins.Question 12.12. The nurse practitioner working in occupational health has been asked to speak to a group of factory workers about the importance of wearing gloves when working with strong chemicals such as turpentine and paint thinner. Which of the following characteristics of cell membranes underlies the nurse's teaching? (Points : 3)Cell membranes are impermeable to all but lipid-soluble substances.Cell membranes have lipids that have a hydrophilic head and a hydrophobic tail.Cell membranes contain receptors for hormones and biologically active substances.Transmembrane proteins can pass through the cell membrane into the intracellular environment.Question 13.13. The NP is providing care for a 21-year-old female patient with gas gangrene of a compound fracture in her arm. Which of the following assessment findings would the nurse most reasonably expect to find when caring for a patient with a diagnosis of gas gangrene? (Points : 3)Inflammation of the affected tissueA positive culture for StaphylococcusSpreading edemaImpaired alveolar gas exchange Question 14.14. A community health nurse practitioner is teaching a group of female high school students about the importance of regular Papanicolaou (Pap) smears. The nurse recognizes that which of the following items underlies the rationale for this teaching? (Points : 3)The active substitution of normal cells in the cervix correlates to cancer risk.Undifferentiated stem cells are an early indicator of cervical cancer.Cancer of the uterine cervix develops incrementally at a cellular level.Dysplasia in the connective tissue of the cervix is a strong precursor to cancer.Question 15.15. Which target of both chemotherapy and radiation treatment accounts for adverse as well as therapeutic effects? (Points : 3)Cell-surface receptorsCirculating hormone levelsBlood vesselsRapidly proliferating cells Question 16.16. A patient who has a diagnosis of lung cancer is scheduled to begin radiation treatment. The NP providing pretreatment education is explaining some of the potential unwanted effects of the treatment. Which of the following statements by the nurse is most accurate? (Points : 3)“Some patients experience longer-term irritation of skin adjacent to the treatment site.”“Sometimes you might find that your blood takes longer to clot than normal.”“The changes that you might see are normally irreversible.”“The unwanted effects will be limited to the exposed portions of your skin.”Question 17.17. The family of a 68-year-old man who is in the end stages of small cell lung cancer is distraught at his visible body wasting that has worsened in recent weeks. Which of the following phenomena best accounts for the patient's anorexia and cachexia? (Points : 3)Inadequate cellular metabolism of glucose results from tumor factorsHigh fat losses coupled with preservation of muscle mass exaggerate the appearance of wastingProducts of the tumor itself as well as a hypermetabolic state cause cachexiaInadequate food intake due to symptoms and treatment results in loss of both muscle and fatQuestion 18.18. The nurse practitioner is seeing a client who has an acute exacerbation of Crohn’s disease. The NP recognizes the fact that the disease involves the inflammation and irritation of the intestinal lining. Which of the following types of tissue is most likely involved in the patient's pathology? (Points : 3)Simple columnar epitheliumGlandular epitheliumSimple cuboidal eptheliumStratified epithelium Question 19.19. Which of the following pregnant women has most likely encountered the greatest increase in the risk that her child will have a fetal anomaly? (Points : 3)A woman with diagnoses of syphilis and cirrhosis of the liverA woman who has herpes simplex and recently recovered from endocarditisA woman with chronic obstructive pulmonary syndrome and tuberculosisA woman with diagnoses of insulin-dependent diabetes mellitus and peripheral neuropathyQuestion 20.20. A nurse practitioner employed in the emergency department admits a patient who has experienced severe frostbite to his hands and toes after becoming lost on a ski trail. The NP recognizes that which of the following phenomena has caused the tissue damage? (Points : 3)Decreased blood viscosity has resulted in interstitial bleeding.Reactive vasodilation has compromised perfusion.Autonomic nervous stimulation has resulted in injury.Decreased blood flow has induced hypoxia.Question 21.21. A nurse practitioner employed in a hospitalist notices that a patient is experiencing muscle atrophy following 2 weeks in traction after a motor vehicle accident. Which of the following factors has most likely contributed to the atrophy of the patient's muscle cells? (Points : 3)High levels of insulin and IGF-1 in the patient's blood during immobilizationDenervation of the affected muscles during the time of tractionA reduction of skeletal muscle use secondary to the traction treatmentReduced oxygen consumption and cellular function that ensures muscle cell survivalQuestion 22.22. An infant who is four days postpartum has been diagnosed with a single-gene disorder. The parents of the child have a number of questions about the etiology of the health problem, which the physician is attempting to address in detail. Which of the following teaching points most accurately captures an aspect of single-gene congenital disorders? (Points : 3)Affected genes are present on autosomal chromosomes rather than sex chromosomes.The majority of single-gene disorders manifest near the time of puberty.A particular defect can be caused by mutations at one of several different loci.Single-gene disorders are associated with existing rather than new mutations.Question 23.23. A researcher is involved in the production of insulin through recombinant DNA technology. Which of the following statements could the researcher best provide as a rationale for her work? (Points : 3)The gene fragment responsible for insulin production can be isolated and reproduced.Particular bacteria are capable of insulin production.It is possible to reproduce the chromosome responsible for insulin production.Recombination of DNA base pairs can result in a gene that will produce insulin.Question 24.24. A 6-year-old girl with a diagnosis of Marfan syndrome is being assessed at a community health clinic. Which of the following assessments would be the health care professional's lowest priority? (Points : 3)A test of the child's visual acuityA musculoskeletal assessmentTests of kidney functionCardiovascular assessment Question 25.25. Following a biopsy, a 54-year-old man has been diagnosed as having a benign neoplastic tumor. Which of the following characteristics most likely applies to his tumor? (Points : 3)The tumor is poorly approximated and has the potential to break loose.The tumor may secrete hormones or cytokines.The well-differentiated neoplastic cells are clustered together in a single mass.It has a rapid rate of growth and can induce ischemia.

1. A 40-year-old woman who experiences severe seasonal allergies has been referred by her family physician to an allergist for weekly allergy injections. The woman is confused as to why repeated exposure to substances that set off her allergies would ultimately benefit her. Which of the following phenomena best captures the rationale for allergy desensitization therapy? (Points : 3)Repeated exposure to offending allergens binds the basophils and mast cells that mediate the allergic response.Exposure to allergens in large, regular quantities overwhelms the IgE antibodies that mediate the allergic response.Repeated exposure stimulates adrenal production of epinephrine, mitigating the allergic response.Injections of allergens simulate production of IgG, which blocks antigens from combining with IgE.

Question 2.2. A 14-year-old boy has been diagnosed with infectious mononucleosis. Which of the following pathophysiological phenomena is most responsible for his symptoms? (Points : 3)The Epstein-Barr virus (EBV) is lysing many of the boy's neutrophils.Viruses are killing some of his B cells and becoming incorporated into the genomes of others.The EBV inhibits the maturation of white cells within his peripheral lymph nodes.The virus responsible for mononucleosis inhibits the maturation of myeloblasts into promyelocytes.Question 3.3. A 66-year-old female patient has presented to the emergency department because of several months of intermittently bloody stools that has recently become worse. The woman has since been diagnosed with a gastrointestinal bleed secondary to overuse of nonsteroidal anti-inflammatory drugs that she takes for her arthritis. The health care team would realize that which of the following situations is most likely?(Points : 3)The woman has depleted blood volume due to her ongoing blood loss.She will have iron-deficiency anemia due to depletion of iron stores.The patient will be at risk for cardiovascular collapse or shock.She will have delayed reticulocyte release. Question 4.4. Which of the following patients is most likely to benefit from transplantation of thymic tissue or major histocompatibility complex (MHC)-compatible bone marrow? (Points : 3)A 12-year-old girl with a history of epilepsy and low IgG levels secondary to phenytoin useA 7-year-old boy whose blood work indicates decreased IgA and IgG with increased IgMA 6-year-old boy whose pre-B cells are incapable of translation to normal B cellsA 9-year-old girl who has a diagnosis of IgA deficiency Question 5.5. A 29-year-old construction worker got a sliver under his fingernail four days ago. The affected finger is now reddened, painful, swollen, and warm to the touch. Which of the following hematological processes is most likely occurring in response to the infection? (Points : 3)Proliferation of immature neutrophilsHigh circulatory levels of myeloblastsIncreased segmented neutrophil productionPhagocytosis by myelocytes Question 6.6. Sputum samples from a patient with pneumonia contain an infective agent that has a peptidoglycan cell wall, expresses endotoxins, replicates readily in broth and on agar, grows in clusters, has pili, and does not stain when exposed to crystal violet. This pneumonia is most likely: (Points : 3)ChlamydialViralMycoplasmalBacterial Question 7.7. A child has been diagnosed with thalassemia. Which of the following other health problems is the child at risk for? (Points : 3)HypocoagulationIron and ferritin deficienciesSplenomegaly and hepatomegalyNeutropenia Question 8.8. A nurse practitioner is providing prenatal care and education for a first-time expectant mother, 22 weeks' gestation, who has a diagnosis of a sexually transmitted infection. Which of the following statements by the expectant mother demonstrates an adequate understanding of vertical disease transmission and congenital infections? (Points : 3)“Gonorrhea and chlamydia pose the greatest risks of transmission from mother to child.”“I know that my baby will need observation for HIV signs and symptoms in the weeks following my delivery.”“My baby could become infected either across the placenta or during the birth itself.”“Prophylactic immunization will reduce my baby's chance of being born with an illness.”Question 9.9. As part of his diagnostic workup, a 77-year-old man's nurse practitioner has ordered blood work that includes ferritin levels. The man is very interested in the details of his health care and is unfamiliar with ferritin and its role. He asks his nurse practitioner to explain the significance of it and the rationale for testing it. Which of the following explanations by the nurse practitioner is most accurate?(Points : 3)“Ferritin is the activated and usable form of iron that your red blood cells can use to transport oxygen.”“Ferritin is a stored form of iron that indirectly shows me whether you would benefit from iron pills.”“Ferritin is a protein-iron complex that allows your red blood cells to make use of the iron that you consume in your diet.”“Ferritin is the form of iron that is transported in your blood plasma to the red blood cells that need it.”Question 10.10. A 16-year-old female has been brought to her primary care nurse practitioner by her mother due to the girl's persistent sore throat and malaise. Which of the following facts revealed in the girl's history and examination would lead the nurse practitioner to rule out infectious mononucleosis? (Points : 3)The girl has a temperature of 38.1°C (100.6°F) and has enlarged lymph nodes.Her liver and spleen are both enlarged.Blood work reveals an increased white blood cell count.Chest auscultation reveals crackles in her lower lung fields bilaterally.Question 11.11. A 30-year-old man has spent 5 hours on a cross-country flight seated next to a passenger who has been sneezing and coughing, and the man has been inhaling viral particles periodically. Which of the following situations would most likely result in the stimulation of the man's T lymphocytes and adaptive immune system? (Points : 3)Presentation of a foreign antigen by a familiar immunoglobulinRecognition of a foreign MHC moleculeRecognition of a foreign peptide bound to a self MHC moleculeCytokine stimulation of a T lymphocyte with macrophage or dendritic cell mediationQuestion 12.12. A 22-year-old female who adheres to a vegan diet has been diagnosed with iron-deficiency anemia. Which of the following components of her diagnostic blood work would be most likely to necessitate further investigation? (Points : 3)Decreased mean corpuscular volume (MCV)Decreased hemoglobin and hematocritMicrocytic, hypochromic red cellsDecreased erythropoietin levels Question 13.13. A couple who are expecting their first child have been advised by friends to consider harvesting umbilical cord blood in order to have a future source of stem cells. The couple have approached their nurse practitioner with this request and are seeking clarification of exactly why stem cells are valuable and what they might expect to gain from harvesting them. How can the nurse practitioner best respond to the couple's inquiry? (Points : 3)“Stem cells can help correct autoimmune diseases and some congenital defects.”“Stem cells can be used to regenerate damaged organs should the need ever arise.”“Stem cells can be used as a source of reserve cells for the entire blood production system.”“Stem cells can help treat some cancers and anemias, but they must come from your child himself or herself.”Question 14.14. A 23-year-old man has received a recent diagnosis of appendicitis following 24 hours of acute abdominal pain. The nurse practitioner providing care for the man is explaining that while it is unpleasant, the inflammation of his appendix is playing a role in his body's fight against the underlying infectious process. Which of the following teaching points should the nurse practitioner eliminate from his teaching for the patient? (Points : 3)“Inflammation can help to remove the body tissue cells that have been damaged by infection.”“Inflammation will start your body on the path to growing new, healthy tissue at the site of infection.“Inflammation helps your body to produce the right antibodies to fight the infection.”“Inflammation ultimately aids in eliminating the initial cause of the cell injury in your appendix.”Question 15.15. A 60-year-old woman is suspected of having non-Hodgkin lymphoma (NHL). Which of the following aspects of her condition would help to rule out Hodgkin lymphoma? (Points : 3)Her neoplasm originates in secondary lymphoid structures.The lymph nodes involved are located in a large number of locations in the lymphatic system.The presence of Reed-Sternberg cells has been confirmed.The woman complains of recent debilitating fatigue. Question 16.16. Which of the following statements most accurately conveys an aspect of lymphatic system activity? (Points : 3)B and T lymphocyte development begins in the bone marrow and ends in the peripheral lymphoid structures.B cells and macrophages are released from the bone marrow in their completed state.Stem cells in the lymph nodes initiate and regulate the process of white cell synthesis.Leukocytes bypass vascular circulation and are distributed instead by the lymphatic system.Question 17.17. A nurse practitioner is explaining to a 40-year-old male patient the damage that Mycobacterium tuberculosis could do to lung tissue. Which of the following phenomena would underlie the nurse practitioner's explanation? (Points : 3)Tissue destruction results from neutrophil deactivation.Nonspecific macrophage activity leads to pulmonary tissue destruction and resulting hemoptysis.Macrophages are unable to digest the bacteria, resulting in immune granulomas.Neutrophils are ineffective against the Mycobacterium tuberculosis antigens.Question 18.18. A 2-year-old girl has had repeated ear and upper respiratory tract infections since she was born. A pediatrician has determined a diagnosis of transient hypogammaglobulinemia of infancy. What is the physiological origin of the child's recurrent infections? (Points : 3)Antibody production by plasma cells is compromised because of impaired communication between B and T cells.The child had a congenital absence of immunoglobulin G (IgG) antibodies and her body is only slowly beginning to produce them independently.The child was born with immunoglobulin A (IgA) and immunoglobulin (IgM) antibodies, suggesting intrauterine infection.The child lacks the antigen presenting cells integral to normal B-cell antibody production.Question 19.19. Following a course of measles, a 5-year-old girl developed scattered bruising over numerous body surfaces and was diagnosed with immune thrombocytopenic purpura (ITP). As part of her diagnostic workup, blood work was performed. Which of the following results is most likely to be considered unexpected by the health care team? (Points : 3)Increased thrombopoietin levelsDecreased platelet count Normal vitamin K levelsNormal leukocyte levels Question 20.20. A 60-year-old male patient with an acute viral infection is receiving interferon therapy. The nurse practitioner is teaching the family of the patient about the diverse actions of the treatment and the ways that it differs from other anti-infective therapies. Which of the following teaching points should the nurse practitioner exclude? (Points : 3)“Interferon can help your father's unaffected cells adjacent to his infected cells produce antiviral proteins that limit the spread of the infection.”“Interferon can help limit the replication of the virus that's affecting your father.”“Interferon helps your father's body recognize infected cells more effectively.”“Interferon can bolster your father's immune system by stimulating natural killer cells that attack viruses.”Question 21.21. A 24-year-old woman presents with fever and painful, swollen cervical lymph nodes. Her blood work indicates neutrophilia with a shift to the left. She most likely has: (Points : 3)A mild parasitic infectionA severe bacterial infectionA mild viral infectionA severe fungal infection Question 22.22. A nurse practitioner student is familiarizing herself with the overnight admissions to an acute medical unit of a university hospital. Which of the following patients would the student recognize as being least likely to have a diagnosis of antiphospholipid syndrome in his or her medical history? (Points : 3)A 66-year-old obese male with left-sided hemiplegia secondary to a cerebrovascular accidentA 90-year-old female resident of a long-term care facility who has been experiencing transient ischemic attacksA 30-year-old female with a diagnosis of left leg DVT and a pulmonary embolismA 21-year-old male with a diagnosis of cellulitis and suspected endocarditis secondary to intravenous drug useQuestion 23.23. The blood work of a 44-year-old male patient with a diagnosis of liver disease secondary to alcohol abuse indicates low levels of albumin. Which of the following phenomena would a clinician be most justified in anticipating? (Points : 3)Impaired immune functionAcid-base imbalancesImpaired thermoregulationFluid imbalances Question 24.24. A nurse practitioner is teaching her colleagues about the role of cytokines in a variety of pathologies. Which of the following teaching points best captures an aspect of the functions and nature of cytokines? (Points : 3)“A particular cytokine can have varied effects on different systems, a fact that limits their therapeutic use.”“Cytokine production is constant over time, but effects are noted when serum levels cross a particular threshold.”“Most cytokines are produced by granular leukocytes, and different cells are capable of producing the same cytokine.”“Cytokine actions are self-limiting in that activation of one precludes activation of other cytokines with similar actions.”Question 25.25. Which of the following phenomena would be least likely to result in activation of the complement system? (Points : 3)Recognition of an antibody bound to the surface of a microbeThe binding of mannose residues on microbial glycoproteinsActivation of Toll-like receptors (TLRs) on complement proteinsDirect recognition of microbial proteins 

1. The nurse practitioner for a cardiology practice is responsible for providing presurgical teaching for patients who are about to undergo a coronary artery bypass graft. Which of the following teaching points best conveys an aspect of the human circulatory system? (Points : 3)“Your blood pressure varies widely between arteries and veins, and between pulmonary and systemic circulation.”“Only around one quarter of your blood is in your heart at any given time.”“Blood pressure and blood volume roughly mimic one another at any given location in the circulatory system.”“Left-sided and right-sided pumping action at each beat of the heart must equal each other to ensure adequate blood distribution.”Question 2.2. A physical assessment of a 28-year-old female patient indicates that her blood pressure in her legs is lower than that in her arms and that her brachial pulse is weaker in her left arm than in her right. In addition, her femoral pulses are weak bilaterally. Which of the following possibilities would her care provider be most likely to suspect? (Points : 3)PheochromocytomaEssential hypertensionCoarctation of the aortaAn adrenocortical disorder Question 3.3. As part of the diagnostic workup for a male patient with a complex history of cardiovascular disease, the care team has identified the need for a record of the electrical activity of his heart, insight into the metabolism of his myocardium, and physical measurements, and imaging of his heart. Which of the following series of tests is most likely to provide the needed data for his diagnosis and care? (Points : 3)Echocardiogram, PET scan, ECGAmbulatory ECG, cardiac MRI, echocardiogramSerum creatinine levels, chest auscultation, myocardial perfusion scintigraphyCardiac catheterization, cardiac CT, exercise stress testingQuestion 4.4. An older adult female patient has presented with a new onset of shortness of breath, and the patient's nurse practitioner has ordered measurement of her BNP levels along with other diagnostic tests. What is the most accurate rationale for the nurse practitioner's choice of blood work? (Points : 3)BNP is released as a compensatory mechanism during heart failure and measuring it can help differentiate the patient's dyspnea from a respiratory pathology.BNP is an indirect indicator of the effectiveness of the RAA system in compensating for heart failure.BNP levels correlate with the patient's risk of developing cognitive deficits secondary to heart failure and consequent brain hypoxia.BNP becomes elevated in cases of cardiac asthma, Cheyne-Stokes respirations, and acute pulmonary edema, and measurement can gauge the severity of pulmonary effects.Question 5.5. A patient in the intensive care unit has a blood pressure of 87/39 and has warm, flushed skin accompanying his sudden decline in level of consciousness. The patient also has arterial and venous dilation and a decrease in systemic vascular resistance. What is this patient's most likely diagnosis? (Points : 3)Hypovolemic shockSeptic shockNeurogenic shockObstructive shock Question 6.6. A number of patients have presented to the emergency department in the last 24 hours with complaints that are preliminarily indicative of myocardial infarction. Which of the following patients is least likely to have an ST-segment myocardial infarction (STEMI)? (Points : 3)A 70-year-old woman who is complaining of shortness of breath and vague chest discomfortA 66-year-old man who has presented with fatigue, nausea and vomiting, and cool, moist skinA 43-year-old man who woke up with substernal pain that is radiating to his neck and jawA 71-year-old man who has moist skin, fever, and chest pain that is excruciating when he moves but relieved when at restQuestion 7.7. A 54-year-old man with a long-standing diagnosis of essential hypertension is meeting with his nurse practitioner. The patient's nurse practitioner would anticipate that which of the following phenomena is most likely occurring? (Points : 3) The patient's juxtaglomerular cells are releasing aldosterone as a result of sympathetic stimulation.Epinephrine from his adrenal gland is initiating the renin-angiotensin-aldosterone system.Vasopressin is exerting an effect on his chemoreceptors and baroreceptors, resulting in vasoconstriction.The conversion of angiotensin I to angiotensin II in his lungs causes increases in blood pressure and sodium reabsorption. Question 8.8. A 66-year-old obese man with a diagnosis of ischemic heart disease has been diagnosed with heart failure that his care team has characterized as attributable to systolic dysfunction. Which of the following assessment findings is inconsistent with his diagnosis? (Points : 3)His resting blood pressure is normally in the range of 150/90 and an echocardiogram indicates his ejection fraction is 30%.His end-diastolic volume is higher than normal and his resting heart rate is regular and 82 beats per minute.He is presently volume overloaded following several days of intravenous fluid replacement.Ventricular dilation and wall tension are significantly lower than normal.Question 9.9. Which of the following assessment findings in a newly admitted 30-year-old male patient would be most likely to cause his nurse practitioner to suspect polyarteritis nodosa? (Points : 3)The man's blood work indicates polycythemia (elevated red cells levels) and leukocytosis (elevated white cells).The man's blood pressure is 178/102 and he has abnormal liver function tests.The man is acutely short of breath and his oxygen saturation is 87%.The man's temperature is 101.9°F and he is diaphoretic (heavily sweating).Question 10.10. A 6-year-old boy has been brought to the emergency department by ambulance after his mother discovered that his heart rate was “so fast I couldn't even count it.” The child was determined to be in atrial flutter and his mother is seeking an explanation from the health care team. Which of the following points should underlie an explanation to the mother? (Points : 3)The child is experiencing a reentry rhythm in his right atrium.The resolution of the problem is dependent on spontaneous recovery and is resistant to pacing interventions.The child is likely to have a normal ECG apart from the rapid heart rate.The boy's atria are experiencing abnormal sympathetic stimulation.Question 11.11. A patient has suffered damage to his pericardium following a motor vehicle accident. Which of the following consequences should the nurse practitioner be most likely to rule out? (Points : 3)Impaired physical restraint of the left ventriculeIncreased friction during the contraction/relaxation cycleReduced protection from infectious organismsImpaired regulation of myocardial contraction Question 12.12. Which of the following situations related to the transition from fetal to perinatal circulation would be most likely to necessitate medical intervention? (Points : 3)Pressure in the pulmonary circulation and the right side of the infant's heart fall markedly.Alveolar oxygen tension increases, causing reversal of pulmonary vasoconstriction of the fetal arteries.Systemic vascular resistance and left ventricular pressure are both increasing.Pulmonary vascular resistance, related to muscle regression in the pulmonary arteries, rises over the course of the infant's first week.Question 13.13. A 70-year-old male patient presents to the emergency department complaining of pain in his calf that is exacerbated when he walks. His pedal and popliteal pulses are faintly palpable and his leg distal to the pain is noticeably reddened. What would his care provider's preliminary diagnosis and anticipated treatment most likely be? (Points : 3)Acute arterial occlusion that will be treated with angioplastyRaynaud disease that will require antiplatelet medicationsAtherosclerotic occlusive disease necessitating thrombolytic therapyGiant cell temporal arteritis that will be treated with corticosteroidsQuestion 14.14. A patient is experiencing impaired circulation secondary to increased systemic arterial pressure. Which of the following statements is the most relevant phenomenon? (Points : 3)Increased preload due to vascular resistanceHigh afterload because of backpressure against the left ventricleImpaired contractility due to aortic resistanceSystolic impairment because of arterial stenosis Question 15.15. A nurse practitioner is providing care for several patients on a medical unit of a hospital. In which of the following patient situations would the nurse practitioner be most likely to rule out hypertension as a contributing factor? (Points : 3)A 61-year-old man who has a heart valve infection and recurrent feverAn 81-year-old woman who has had an ischemic stroke and has consequent one-sided weaknessA 44-year-old man awaiting a kidney transplant who requires hemodialysis three times per weekA 66-year-old woman with poorly controlled angina and consequent limited activity toleranceQuestion 16.16. A formerly normotensive woman, pregnant for the first time, develops hypertension and headaches at 26 weeks' gestation. Her blood pressure is 154/110 mm Hg and she has proteinuria. What other labs should be ordered for her? (Points : 3)Plasma angiotensin I and II and reninUrinary sodium and potassiumPlatelet count, serum creatinine, and liver enzymesUrinary catecholamines and metabolites Question 17.17. A nurse practitioner is instructing a group of older adults about the risks associated with high cholesterol. Which of the following teaching points should the participants try to integrate into their lifestyle after the teaching session? (Points : 3)“Remember, the 'H' in HDL and the 'L' in LDL correspond to high danger and low danger to your health.”“Having high cholesterol increases your risk of developing diabetes and irregular heart rate.”“Smoking and being overweight increases your risk of primary hypercholesterolemia.”“Your family history of hypercholesterolemia is important, but there are things you can do to compensate for a high inherited risk.”Question 18.18. An autopsy is being performed on a 44-year-old female who died unexpectedly of heart failure. Which of the following components of the pathologist's report is most suggestive of a possible history of poorly controlled blood pressure? (Points : 3)“Scarring of urethra suggestive of recurrent urinary tract infections is evident.”“Bilateral renal hypertrophy noted.”“Vessel wall changes suggestive of venous stasis are evident.”“Arterial sclerosis of subcortical brain regions noted.” Question 19.19. A nurse practitioner has ordered the measurement of a cardiac patient's electrolyte levels as part of the patient's morning blood work. Which of the following statements best captures the importance of potassium in the normal electrical function of the patient's heart? (Points : 3)Potassium catalyzes the metabolism of ATP, producing the gradient that results in electrical stimulation.Potassium is central to establishing and maintaining the resting membrane potential of cardiac muscle cells.The impermeability of cardiac cell membranes to potassium allows for action potentials achieved by the flow of sodium ions.The reciprocal movement of one potassium ion for one sodium ion across the cell membrane results in the production of an action potential.Question 20.20. In which of the following patient situations would a nurse practitioner be most justified in preliminarily ruling out pericarditis as a contributing pathology to the patient's health problems? (Points : 3)A 61-year-old man whose ECG was characterized by widespread T wave inversions on admission but whose T waves have recently normalizedA 77-year-old with diminished S3 and S4 sounds, an irregular heart rate, and a history of atrial fibrillationA 56-year-old obese man who is complaining of chest pain that is exacerbated by deep inspiration and is radiating to his neck and scapular ridgeA 60-year-old woman whose admission blood work indicates elevated white cells, erythrocyte sedimentation rate, and C-reactive protein levelsQuestion 21.21. During a routine physical examination of a 66-year-old woman, her nurse practitioner notes a pulsating abdominal mass and refers the woman for further treatment. The nurse practitioner is explaining the diagnosis to the patient, who is unfamiliar with aneurysms. Which of the following aspects of the pathophysiology of aneurysms would underlie the explanation the nurse provides? (Points : 3)Aneurysms are commonly a result of poorly controlled diabetes mellitus.Hypertension is a frequent modifiable contributor to aneurysms.Individuals with an aneurysm are normally asymptomatic until the aneurysm ruptures.Aneurysms can normally be resolved with lifestyle and diet modifications.Question 22.22. A 66-year-old patient's echocardiogram reveals a hypertrophied left ventricle, normal chamber volume, and a normal ejection fraction from the heart. What is this patient's most likely diagnosis? (Points : 3)Mitral valve regurgitationAortic valve stenosisMitral valve stenosisAortic valve regurgitation Question 23.23. A nurse practitioner is teaching a student NP about the physiologic basis for damage to the circulatory and neurological systems that can accompany hypotension. Which of the following responses by the student would warrant correction by the nurse practitioner? (Points : 3)“As vessel wall thickness increases, tension decreases.”“Smaller blood vessels require more pressure to overcome wall tension.”“The smaller the vessel radius, the greater the pressure needed to keep it open.”“Tension and vessel thickness increase proportionately.” Question 24.24. An 81-year-old female patient of a long-term care facility has a history of congestive heart failure. The nurse practitioner caring for the patient has positioned her sitting up at an angle in bed and is observing her jugular venous distention. Why is jugular venous distention a useful indicator for the assessment of the patient's condition? (Points : 3)Increased cardiac demand causes engorgement of systemic blood vessels, of which the jugular vein is one of the largest.Blood backs up into the jugular vein because there are no valves at the point of entry into the heart.Peripheral dilation is associated with decreased stroke volume and ejection fraction.Heart valves are not capable of preventing backflow in cases of atrial congestion.Question 25.25. A 22-year-old male is experiencing hypovolemic shock following a fight in which his carotid artery was cut with a broken bottle. What immediate treatments are likely to most benefit the man? (Points : 3)Resolution of compensatory pulmonary edema and heart arrhythmiasInfusion of vasodilators to foster perfusion and inotropes to improve heart contractilityInfusion of normal saline of Ringer lactate to maintain the vascular spaceAdministration of oxygen and epinephrine to promote perfusion

1. Which of the following patients who presented to a walk-in medical clinic is most likely to be diagnosed with a rhinosinusitis rather than a common cold? (Points : 3)A man complaining of general fatigue, a headache, and facial pain with a temperature of 100.9°FA woman presenting with malaise, lethargy, and copious nasal secretionsA man with a dry, stuffy nasopharynx, a sore throat, and temperature of 98.9°FA woman complaining of generalized aches who has a hoarse voice and reddened, painful upper airwaysQuestion 2.2. A 44-year-old woman developed calf pain during a transatlantic flight followed by acute shortness of breath upon arrival at her destination. She was subsequently diagnosed with a pulmonary embolism (PE), which resolved with anticoagulant therapy. Which of these statements best characterizes the underlying problem of her PE? (Points : 3)She was short of breath because ventilation was occurring but perfusion was inadequate.The combination of normal perfusion but compromised ventilation caused hypoxia.She developed a transient anatomic shunt resulting in impaired oxygenation.Impaired gas diffusion across alveolar membranes resulted in dyspnea and hypoxia.Question 3.3. A premature infant on mechanical ventilation has developed bronchopulmonary dysplasia (BPD) and is showing signs and symptoms of hypoxemia, low lung compliance, and respiratory distress syndrome (RDS). Which of the following is the most likely contributor to the infant's present health problem? (Points : 3)High inspired oxygen concentration and injury from positive-pressure ventilationFailure to administer corticosteroids to the infant in uteroInsufficient surfactant production and insufficient surfactant therapyInsufficient supplemental oxygen therapy Question 4.4. Which of the following residents of a long-term care facility is most likely to be exhibiting the signs and symptoms of chronic obstructive pulmonary disease (COPD)? (Points : 3)A 79-year-old lifetime smoker who is complaining of shortness of breath and pain on deep inspirationAn 81-year-old smoker who has increased exercise intolerance, a fever, and increased white blood cellsAn 81-year-old male who has a productive cough and recurrent respiratory infectionsAn 88-year-old female who experiences acute shortness of breath and airway constriction when exposed to tobacco smokeQuestion 5.5. A 66-year-old male presents to the emergency department accompanied by his wife who claims that he has been acting confused. The man is complaining of a sudden onset of severe weakness and malaise and has a dry cough and diarrhea. His temperature is 102.8°F and his blood work indicates his sodium level is 126 mEq/L (normal is 135 to 145 mEq/L). What will be the assessing nurse practitioner's most likely suspicion? (Points : 3)BronchopneumoniaMycoplasma pneumoniaLegionnaire diseasePneumococcal pneumonia Question 6.6. A 51-year-old female patient who is 2 days postoperative on a surgical unit of a hospital is at risk for developing atelectasis as a result of being largely immobile. Which of the following teaching points by her nurse practitioner is most appropriate? (Points : 3)“Being in bed increases the risk of fluid accumulating between your lungs and their lining, so it's important for you to change positions often.”“You should breathe deeply and cough to help your lungs expand as much as possible while you're in bed.”“Make sure that you stay hydrated and walk as soon as possible to avoid our having to insert a chest tube.”“I'll prescribe bronchodilator medications that will help open up your airways and allow more oxygen in.”Question 7.7. Due to complications, a male postoperative patient has been unable to mobilize for several days following surgery and has developed atelectasis. Which of the following processes would his care team anticipate with relation to his health problem? (Points : 3)Vasodilation in the alveolar vessels in the affected region of his lungIncreased workload for the left side of the patient's heartIncreased blood flow to the area of atelectasisRedirection of blood flow away from the lung regions that are hypoxicQuestion 8.8. Which of the following clinical findings would be most closely associated with a patient who has interstitial lung disease rather than COPD? (Points : 3)Audible wheezing on expirationDiminished expiratory flow ratesIncreased respiratory rate with decreased tidal volumeNormal compliance of alveolar tissue Question 9.9. A male lifetime smoker has died as a result of chronic obstructive pulmonary disease. Which of the following phenomena regarding his alveoli would his care team have most reasonably expected in the weeks prior to his death? (Points : 3)Proliferation of natural killer (NK) cells in the alveolar lumenLarge numbers of alveolar macrophages in septal connective tissueThe presence of tubercles in the intra-alveolar spacesCompensatory regeneration of type I alveolar cells Question 10.10. A 21-year-old male patient has suffered a head injury during a crash on his motorcycle, and a deficit that assessments have revealed is an impaired swallowing mechanism. He has also developed aspiration pneumonia. Which of the following statements most accurately capture an aspect of his condition? (Points : 3)His oropharynx is obstructed.His epiglottis is covering his larynxHis vocal folds have been compromised.His tracheobronchial tree is intermittently obstructed. Question 11.11. A short, nonsmoking middle-aged man presents to the emergency department with left-sided chest pain and a cough. He says that the pain started abruptly, just after lunch, and that breathing and coughing make it worse. He denies recent injury. He is breathing shallowly and rapidly and expresses fear that he may be having a heart attack. Breath sounds are normal, and he is not cyanotic. Which condition is most likely causing his symptoms? (Points : 3)Myocardial infarctionSpontaneous pneumothoraxPleuritis related to infectionObstructive atelectasis Question 12.12. Which of the following statements best conveys an aspect of the respiratory pressures that govern ventilation? (Points : 3)Intrapleural pressure slightly exceeds that of the inflated lungThe chest wall exerts positive pressure on the lungs that contributes to expirationThe lungs are prevented from collapsing by constant positive intrapulmonary pressureNegative intrapleural pressure holds the lungs against the chest wallQuestion 13.13. The mother of a 7-year-old boy who has recently been diagnosed with childhood asthma has come to the education center to learn more about her son's condition. Which of the following teaching points is most justifiable? (Points : 3)“Research has shown that viruses may actually be a factor in many children's asthma.”“The most reliable indicator that your child is having an asthma attack is audible wheezing.”“Steroids that your child can inhale will likely be the first line of defense.“Your son will likely need to limit or avoid exercise and sports.”Question 14.14. Which of the following phenomena is most likely occurring during a child's alveolar stage of lung development? (Points : 3)Terminal alveolar sacs are developing and surfactant production is beginning.A single-capillary network exists and the lungs are capable of respiration.The conducting airways are formed, but respiration is not yet possible.Primitive alveoli are formed and the bronchi and bronchioles become much larger.Question 15.15. A 71-year-old woman is dependent on oxygen therapy and bronchodilators due to her diagnosis of emphysema. Which of the following processes would her care team be most justified in ruling out? (Points : 3)Decreased elastic recoil due to alveolar damageDecreased residual lung volume due to impaired alveolar ventilationIncreased anatomic dead space due to reduced tidal volumeIncreased alveolar dead space due to incorrect intrapleural pressureQuestion 16.16. A definitive test for cystic fibrosis is (Points : 3)The sweat testA sputum cultureA fecal fat testA Chymex test for pancreatic insufficiency Question 17.17. A patient who presented with shortness of breath and difficulty climbing stairs has been diagnosed with pulmonary fibrosis, a disease characterized by scarring of the alveoli. What would her care team anticipate when observing her breathing? (Points : 3)Rapid, deep breathsWheezing breathsShort, shallow breathsPursed-lip breaths Question 18.18. Which of the following patients are NOT displaying known risk factors for the development of pulmonary emboli? (Points : 3)A patient who is immobilized following orthopedic surgeryA patient who has impaired Cl– and Na+ regulationA patient who is taking amiodarone for the treatment of a cardiac arrhythmiaA patient who is a smoker and takes oral contraceptivesA patient who is undergoing radiation therapy for the treatment of breast cancerQuestion 19.19. Following a winter power outage, a patient who had been using a home gasoline generator began to experience dizziness and headaches and was diagnosed with carbon monoxide poisoning. What is the goal of hyperbaric oxygen treatment for carbon monoxide poisoning? (Points : 3)To increase the amount of oxygen dissolved in plasmaTo increase the production of unbound hemoglobinTo stimulate the release of oxygen at the capillariesTo remove bound CO from hemoglobin Question 20.20. A nurse practitioner is performing patient teaching about the influenza virus with each patient who has come to the clinic to receive that year's vaccine. Which of the following statements by patients best reflects an accurate understanding of the flu virus? (Points : 3)“I could come down with viral or bacterial pneumonia as a result of a bad flu bug.”“I know my vaccination is especially important since there aren't any drugs that can treat the flu once I get sick with it.”“The emphasis on bundling up, staying warm, and drinking lots of fluids is outdated and actually ineffective.”“Like all vaccines, it is ideal if everyone in a population gets immunized against the flu.”Question 21.21. A 77-year-old lifetime smoker has been diagnosed with a tumor in his lung at the site of an old tubercle scarring site, located in a peripheral area of his bronchiolar tissue. What is this patient's most likely diagnosis? (Points : 3)Squamous cell carcinomaSmall cell lung cancerLarge cell carcinomaAdenocarcinoma Question 22.22. As a result of dehydration, a patient's epithelial cells are producing insufficient amounts of mucus. Consequently, the patient's mucociliary blanket is compromised. Which of the following changes would a care provider most reasonably anticipate as a direct result of this change? (Points : 3)Impaired function of the patient's ciliaDecreased levels of oxygen saturationIncreased amounts of bacteria in the lungsIncreased carbon dioxide levels Question 23.23. Which of the following statements best captures the etiology of the acute response phase of extrinsic (atopic) asthma? (Points : 3)IgG production is heightened as a consequence of exposure to an allergen.Airway remodeling results in airflow limitations.Epithelial injury and edema occur along with changes in mucociliary function.Chemical mediators are released from presensitized mast cells.Question 24.24. A 62-year-old female smoker is distraught at her recent diagnosis of small cell lung cancer (SCLC). How can her nurse practitioner most appropriately respond to her? (Points : 3)“I'm sure this is very hard news to hear, but be aware that with aggressive treatment your chances of beating this are quite good.”“This is very difficult to hear, I'm sure, and we have to observe to see if it spreads because that often happens.”“I'm very sorry to have to give you this news; I'd like to talk to you about surgical options, however.”“This is a difficult diagnosis to receive, but there is a chance that the cancer may go into remission.”Question 25.25. About 3 weeks after razing an old chicken house, a 71-year-old retired farmer has developed a fever, nausea, and vomiting. After ruling out more common health problems, his care provider eventually makes a diagnosis of histoplasmosis. Which of the following processes is most likely taking place? (Points : 3)Toxin production by Histoplasma capsulatum is triggering an immune response.Antibody production against the offending fungi is delayed by the patient's age and the virulence of the organism.Spore inhalation initiates an autoimmune response that produces the associated symptoms.Macrophages are able to remove the offending fungi from the bloodstream but can't destroy them.

1. A 60-year-old man has been diagnosed with renal calculi after repeated episodes of excruciating flank pain in recent weeks. The man states, “I don't know how this could happen to me, since I'm so careful about eating a healthy diet.” What is the most appropriate response to the man's statement? (Points : 3)“Your diet may have played a part in this, but in fact genetics is likely primarily to blame.”“What you eat can influence your risk of stone formation, but many other factors like hormones and your metabolism are involved.”“You likely don't need to change your diet, but now that you have stones in one kidney, you're at very high risk of growing them in the other kidney.”“Your diet might be normally healthy, but high intake of normally beneficial minerals like calcium and magnesium can lead to stones.”Question 2.2. An 87-year-old male resident of an assisted living facility has been consistently continent of urine until the last several weeks. Which of the following actions by the care providers at the facility is the most likely priority? (Points : 3) Performing a physical examination and history to determine the exact cause and character of the incontinenceProviding patient education focusing on the fact that occasional incontinence is a normal, age-related changeTeaching the resident about protective pads, collection devices, and medications that may be effectiveShowing the resident the correct technique for exercises to improve bladder, sphincter, and pelvic floor toneQuestion 3.3. A 34-year-old male patient has diagnoses of liver failure, ascites, and hepatic encephalopathy secondary to alcohol abuse. The patient's family is questioning the care team about why his abdomen is so large even though he is undernourished and emaciated. Which of the following statements most accurately underlies the explanation that a member of the care team would provide to the family? (Points : 3)An inordinate amount of interstitial fluid is accumulating in the patient's abdomen.The transcellular component of the intracellular fluid compartment contains far more fluid than normal.The normally small transcellular fluid compartment, or third space, is becoming enlarged.Gravity-dependent plasma is accumulating in the patient's peritoneal cavity.Question 4.4. A patient is brought to the emergency department with complaints of shortness of breath. Assessment reveals a full, bounding pulse, severe edema, and audible crackles in the lower lung fields bilaterally. What is the patient's most likely diagnosis? (Points : 3)HyponatremiaFluid volume excessHypocalcemiaHyperkalemia Question 5.5. Following several days in an acidotic state, a hospital patient has returned to the desired pH. Which of the following processes could have contributed to the resolution of the patient's health problem?(Points : 3)Exchange of Na+ and H+ ionsSelective renal secretion and reabsorption of CO2Phosphate and ammonia buffer systems in the renal tubulesExcretion of HCO3– by the kidneys Question 6.6. Which of the following patients would be considered to have a significant risk of developing the prerenal form of acute renal failure? (Points : 3)A 22-year-old male who has lost a large amount of blood following a workplace injuryA 41-year-old female who is admitted for intravenous antibiotic treatment of pyelonephritisA 79-year-old male with diagnoses of poorly controlled diabetes mellitus and congestive heart failureA 20-year-old male who is admitted for treatment of an overdose of a nephrotoxic drugA 68-year-old male with a diagnosis of benign prostatic hyperplasia (BPH)An 80-year-old female who has been admitted for treatment of dehydration, hyponatremia, and malnutrition Question 7.7. An 81-year-old female has long-standing hypocalcemia secondary to kidney disease and will shortly be moving into an assisted living facility from her own apartment. Which of the following findings should staff at the facility be instructed to observe for? (Points : 3)Loss of appetite and complaints of nauseaMuscular spasms and complaints of crampsHigh fluid intake and urine outputLethargy and stuporQuestion 8.8. A 55-year-old man has made an appointment to see his nurse practitioner because he has been awakening three to four times nightly to void and often has a sudden need to void with little warning during the day. What is the man's most likely diagnosis and possible underlying pathophysiologic problem? (Points : 3)Stress incontinence due to damage to CNS inhibitory pathwaysOveractive bladder that may result from both neurogenic and myogenic sourcesOveractive bladder due to intravesical pressure exceeding urethral pressureOverflow incontinence that can result from displacement of the angle between the bladder and the posterior proximal urethraQuestion 9.9. A 68-year-old woman with a new onset of vascular dementia has recently begun retaining urine. Which of the following physiological phenomena would her care providers most realistically expect to be currently occurring as a result of her urinary retention? (Points : 3)Hypertrophy of the bladder muscle and increased bladder wall thicknessDecreased urine production and nitrogenous wasted excretion by the kidneysDecompensation, bladder stretching, and high residual urine volumeOverflow incontinence and loss of contraction powerQuestion 10.10. Which of the following individuals is at the highest risk for developing a urinary tract infection (UTI)? (Points : 3)A 60-year-old man with a history of cardiovascular disease who is recovering in the hospital from a coronary artery bypass graftA 66-year-old man undergoing dialysis for the treatment of chronic renal failure secondary to hypertensionA 38-year-old man with high urine output due to antidiuretic hormone insufficiencyA 30-year-old obese woman with poorly controlled diabetes mellitusQuestion 11.11. A nurse educator is orientating new nurses to a renal unit of a hospital. Which of the following teaching points should the nurse include as part of a review of normal glomerular function? (Points : 3)“Nephrons are delicate structures that cannot endure the high pressure that exists in capillary beds elsewhere in the body.”“Glomerular filtrate is very similar in composition to blood plasma found elsewhere in circulation.”“Dilation of the afferent arteriole allows more blood into the nephron and increases the glomerular filtration rate.”“The glomerulus is located between an arteriole and a venule that work together to regulate blood flow.”Question 12.12. A 51-year-old woman diagnosed with multiple sclerosis (MS) five months prior is distressed that she has had several recent episodes of urinary incontinence. She has asked her nurse practitioner why this is the case. Which of the following statements best captures the facts that would underlie the nurse's response to the patient? (Points : 3)Neurologic diseases like MS often result in flaccid bladder dysfunction.She may be unable to sense her bladder filling as a result of her MS.Lesions of the basal ganglia or extrapyramidal tract associated with MS inhibit detrusor contraction.Pathologic reductions in bladder volume brought on by MS necessitate frequent micturition.Question 13.13. A patient with poorly controlled diabetes mellitus presents to the emergency department with suspected ketoacidosis. Which of the following diagnostic results would be most likely to confirm this diagnosis? (Points : 3)Low O2 levels, increased anion gap, base excessHigh ammonia levels, decreased anion gap, high potassiumIncreased CO2, increased anion gap, base deficitDecreased CO2, decreased anion gapQuestion 14.14. A pediatric unit will be receiving from a rural medical outpost a patient transfer of an 8-day-old infant with a suspected congenital renal disorder. Which of the following possibilities is the care team most likely to be able to rule out early? (Points : 3)One of the infant's kidneys may have failed to develop to a normal size.The kidneys may be misshapen and have cysts.The upper or lower poles of the two kidneys may be fused.Renal cell carcinoma may be present.Question 15.15. A 4-year-old boy who has been deaf since birth and has bilateral cataracts has been brought to the emergency department by his mother because she noticed blood in the toilet after he last voided. Urinalysis confirms heavy microscopic hematuria as well as proteinuria. What will the care team's initial differential diagnosis most likely be? (Points : 3)Alport syndromeSystemic lupus erythematosus glomerulonephritisHenoch-Schonlein purpura nephritisImmunoglobulin A nephropathyQuestion 16.16. Which of the following individuals would be considered to be at risk for the development of edema?(Points : 3)An 81-year-old man with right-sided heart failure and hypothyroidismA 60-year-old obese female with a diagnosis of poorly controlled diabetes mellitusA 34-year-old industrial worker who has suffered extensive burns in a job-related accidentA 77-year-old woman who has an active gastrointestinal bleed and consequent anemiaA 22-year-old female with hypoalbuminemia secondary to malnutrition and anorexia nervosaQuestion 17.17. A 22-year-old female with a history of intermittent flank pain, repeated UTIs, and hematuria has been diagnosed with autosomal dominant polycystic kidney disease (ADPKD). Which of the following phenomena has most likely contributed to the development of her health problem? (Points : 3)UTIs coupled with an impaired immune response have caused her ADPKD.She has inherited a tendency for epithelial cell in her tubules to proliferate inappropriately.Severe hypertension and portal hypertension are likely precursors.She has inherited undersized kidneys that are prone to calculi formation.Question 18.18. A 31-year-old patient with a diagnosis of end-stage liver failure has been admitted to the intensive care unit of a hospital. Arterial blood gas sampling indicates that the man has an acid-base imbalance. Which of the following situations is most likely to result in an inappropriate pH? (Points : 3)Conservation or formation of new HCO3– by the kidneysLow albumin and plasma globulin levelsTranscompartmental exchange of H+ and potassium ionsRenal excretion of HCO3– in the presence of excess baseQuestion 19.19. Which of the following data would a clinician consider to be most indicative of acute renal failure? (Points : 3)Alterations in blood pH, peripheral edemaIncreased nitrogenous waste levels, decreased glomerular filtration rate (GFR)Decreased serum creatinine and

Show more
  • @
  • 177 orders completed
ANSWER

Tutor has posted answer for $65.00. See answer's preview

$65.00

******** ** * student nurse ************ **** her ********* ***** the ******* ** different ******* *** ***** cellular origins ****** *** ******* of *********** ***** of *** ********* ********** ** *** preceptor **** describes *** ******* ** cell differentiation? ******* * ********** ** *** hematopoietic system produce *** appropriate **** ***** that *** ******** ** each ***** ** ****************** single **** **** ************** **** approximately *** ********* ***** ** ************ ********** **** undergoes a series ** ********* ******** many different **** types”“Cells differentiate **** necessary **** ***** ******* ***** conception *** ceasing **** *** time ** **************** ** * *********** **** ******* with * ********* ** stomach cancer *** been found ** **** ********** ** *** ***** The ******* *** *** ****** *** surprised ** **** **** ** ****** stating that **** don't *** *** ** ***** have ********* ****** in *** ***** Which ** the following ***** would ******** the ***** that the **** **** provides? ******* * ***** *********** tissue of the ***** ** ************ susceptible ** ********* *************** portal circulatory ****** ****** ****** blood **** *** gastrointestinal tract into *** ************ ******* ****** ****** *************** with ********* cells ********* **** ** anchorage ************* proximity of *** liver ** *** stomach ****** *** ****** ****** of ********* ***** *** ** * **** of ******* inhibitionQuestion ** The ** ** teaching * group ** older adults ***** *** ***** ** ********* ***** ********** ************ ** ***** **** ***** ** the following ********** **** captures the ********* ********** the *** ************** * ************** ******* *** ******* ** ******** oxygen ******* (ROS)Antioxidants ******* the formation ** superoxide ********************* ***** *************** **** moleculesAntioxidants ******* *** occurrence of **** ********* ******** ** *** ***** ************ ** providing **** for * ******* **** * ********* ** ********* *** he notes **** *** ********* sclerae *** jaundiced *** ***** ************ ******* **** ******** is ****** ** excess ************ ** bilirubin * pigment **** *** ********** ** ***** **** ** *** ***** ******* : *********************** ************** *********** ********* **** Question 55 ***** ** the following ******** ** * ******* **** ***** practitioner ***** not require extra screening *** cancer? ******* : *** *********** woman ***** *********** **** ** ****** ******* *********** *** who takes ***************** drugs ********* a ****** transplantA *********** male *** ** ***** *** *** a low-fiber high-fat ***** *********** female **** Down syndrome and ********** ***************** ** * *** older ****** patient ** * ********* care ******** *** * ********* ** **** 1 neurofibromatosis ****** ** **** ** the intake ********** ******** *** *** ******** *** ******** educator is teaching *** **** ***** ***** *** diagnosis ***** ** *** ********* ********** **** ********** conveys ** ****** ** neurofibromatosis? (Points : ******** ************ lesions are unsightly *** the ******* but they are *** ************* ********* puts her ** higher **** ** ********** a ********* neoplasm”“She is ****** **** an ******* of ** ********* ********* ***************** ******* is ****** ** be ************** as * ****** ** *** disease”Question ** * ***** ********* * ***** **** is *** ****** ** *** *********** of two ********* ***** neither ** ***** could **** ******** *** trait independently ***** of *** following ************ **** captures *** ******* *********** for ***** ******* * ***** trait ** ** expression ** multiple allelesEpistasis *** ******** *** phenotypic ********** phenomenon ** an example ** polygenic ************** ******* is *** result of *** *********** ******* ************* ************* ** ***** of *** ********* ********** most accurately ******* an ****** of **** ****** due to ******** ******* homeostasis? ******* * ******** ************* ******* *** levels are ****** **** ************* ************** *** ******* toxins ***** a ******** ** ********* ************** ***** **** ** accumulate ********** ******* ****** cause ** ********** ** damaging *************** ** * group of researchers has identified that *** ********** ** *** ********** ******* ********* share a *********** *********** ***** ** *** following ********** **** ******* *** ******* ********* *** **** situation? ******* * 3)There ** likely * **************** ************ between *** *** genes ************** *********** ********** **** **** *** ****** ******* *********** genes ******* **** ******** *** ****** in *** same section ** *** same ************* ********* ****** ***** *** **** ***** Question **** A **** ******* of * ***** ************ *** ** autosomal ******** ******** The patient *** *** ******* *** *********** ******** * ****** ***** of *** patient's following ********** indicates *** ******* *** ** ******** understanding of the genetic ***** ** this ****** ******** ******* : ****** know there's ** *** of accurately *********** *** ****** **** ** ***** **** inherit *** *************** ******** who ***** have *** disease ***** *** the risk ** ******* it ** ** their children”“I **** **** *** ******* ********* won't ***** ******* ****************** **** that * ****** ****** ****** is ** blame *** *** ****** ****************** **** ** **** ** ** *********** ** * ******* ********** ******** * ***** of ******* ******** ** *************** ******* autosomal recessive ********* *** ********* ******** ********* ***** of *** ********* ********** ** true of ********* ********* ********** ******* * 3)They *** manifest **** ******* ** *** ** **** gene ********** ** * *** in two ****** ** ** affected child ** **** ********* with ** ******** ********** **** ** **** * **** ******* symptomatology **** autosomal ******** ************ associated ********* *** ******* ************ ** abnormalities ** structural **************** 1212 The nurse ************ ******* in occupational ****** has **** asked ** ***** ** a ***** ** ******* workers about the ********** ** ******* ****** when ******* with strong ********* **** ** ********** *** ***** ******* ***** ** the ********* characteristics ** **** ********* underlies *** ******* ********* ******* : 3)Cell ********* *** impermeable ** all *** ************* ************** ********* **** ****** **** **** * hydrophilic **** *** * *********** tailCell membranes contain ********* *** ******** and biologically ****** *********************** proteins *** **** ******* the cell ******** **** the ************* ******************* **** *** ** ** ********* **** for a *********** ****** patient with *** gangrene of * compound ******** in *** arm ***** ** *** ********* ********** ******** ***** *** ***** **** ********** ****** ** **** **** caring for * patient **** * ********* of *** ********* ******* : ************** ** *** affected ******* ******** ******* *** *********************** edemaImpaired ******** *** ******** ******** **** * community health ***** ************ is ******** * ***** ** ****** high school ******** ***** *** ********** ** ******* ************ (Pap) smears *** ***** ********** that which of *** ********* ***** ********* *** ********* for **** ********* ******* * 3)The ****** substitution ** ****** ***** ** *** ****** ********** ** ****** ******************** stem ***** *** ** early ********* ** cervical ************ of the ******* ****** develops ************* at * ******** ************** ** *** connective ****** ** the cervix ** a strong ********* to cancerQuestion 1515 ***** target ** **** ************ and ********* treatment ******** *** ******* ** well ** *********** ******** ******* * ************** receptorsCirculating hormone *********** ************** proliferating ***** Question 1616 * ******* who *** a ********* ** **** ****** ** ********* to begin ********* treatment The NP ********* pretreatment ********* is ********** **** of *** ********* ******** effects ** *** ********* Which ** *** following ********** ** *** ***** is **** ********* ******* * ********* patients ********** longer-term ********** ** skin ******** ** *** ********* site”“Sometimes you ***** find **** **** ***** ***** ****** ** **** than normal”“The ******* **** *** ***** see are ******** ********************* ******** ******* **** ** ******* to *** ******* portions ** **** *************** **** *** family ** * *********** *** *** is ** the *** ****** ** small **** lung ****** ** ********** ** *** visible **** ******* **** *** ******** in ****** ***** ***** ** the ********* ********* best ******** for *** ********* ******** *** cachexia? ******* * ************ cellular metabolism ** ******* results from ***** factorsHigh *** ****** ******* **** preservation of ****** **** exaggerate the ********** ** *************** ** the tumor ****** ** **** ** * ************** ***** ***** cachexiaInadequate **** ****** *** ** ******** *** ********* ******* ** **** of **** ****** *** fatQuestion 1818 *** ***** ************ is ****** * client who *** ** acute exacerbation ** ********* ******* *** NP ********** the **** that *** ******* involves the ************ *** ********** ** the ********** ****** ***** ** *** ********* types ** ****** is **** ****** ******** in *** ********* pathology? (Points * ******** ******** ******************* epitheliumSimple cuboidal eptheliumStratified ********** ******** 1919 ***** of *** ********* ******** ***** *** **** ****** *********** *** ******** ******** ** *** risk **** *** ***** **** **** * fetal ******** ******* : *** woman **** ********* ** ******** *** ********* of *** ****** woman *** has ****** ******* *** ******** ********* **** ************* woman **** chronic *********** ********* ******** and ************* ***** **** diagnoses ** ***************** ******** mellitus *** ********** ****************** 2020 * ***** ************ ******** ** *** ********* ********** ****** * patient *** *** *********** ****** ********* ** *** hands and toes ***** becoming **** ** a *** trail *** ** ********** that ***** ** *** following ********* *** ****** *** ****** ******* ******* * 3)Decreased ***** ********* *** resulted ** ************ **************** ************ *** compromised ****************** ******* *********** *** ******** ** injuryDecreased ***** **** has ******* hypoxiaQuestion 2121 A ***** practitioner employed ** * *********** ******* **** a ******* ** experiencing ****** ******* ********* * weeks in traction ***** * ***** vehicle ******** ***** ** *** following ******* *** **** ****** *********** ** *** atrophy ** the patient's ****** ****** ******* : ****** ****** of ******* *** ***** ** *** patient's ***** ****** ************************* of the ******** ******* ****** the **** ** ********* ********* ** ******** muscle *** ********* ** the ******** **************** ****** *********** *** ******** function that ******* ****** **** survivalQuestion 2222 ** infant *** ** **** **** ********** *** been diagnosed **** * *********** ******** The ******* of *** ***** have * ****** ** questions ***** *** etiology ** *** health ******* which *** ********* ** ********** ** ******* ** detail ***** of the ********* ******** ****** **** accurately ******** ** ****** ** single-gene ********** disorders? (Points : ********** ***** *** present ** autosomal *********** ****** **** *** ************** majority ** *********** ********* manifest **** *** **** ** ******** ********** defect *** be ****** ** ********* ** *** ** ******* ********* *************** disorders *** ********** with ******** ****** **** new ***************** **** A ********** ** ******** in *** production of insulin ******* *********** *** ********** ***** of *** ********* ********** could *** ********** **** ******* ** a ********* *** her work? (Points * 3)The gene ******** *********** for ******* ********** *** be ******** and ******************** ******** are ******* ** ******* productionIt ** possible ** ********* *** ********** responsible *** ******* *********************** ** *** base ***** can ****** ** * **** **** **** produce *************** **** * ********** **** **** * diagnosis of ****** ******** ** being assessed at * ********* ****** clinic ***** ** *** ********* assessments would ** the ****** care professional's lowest priority? ******* : *** **** ** *** child's ****** ******* musculoskeletal *************** ** ****** functionCardiovascular assessment ******** **** ********* * biopsy * *********** man *** been ********* ** ****** * ****** ********** ***** ***** ** *** following *************** most ****** ******* ** *** ****** ******* * ***** ***** ** ****** ************ *** has *** ********* ** ***** ******** ***** *** secrete hormones ** ************ well-differentiated ********** cells *** clustered ******** ** * ****** massIt *** * ***** **** ** ****** and *** induce ********* A 40-year-old ***** *** *********** ****** seasonal ********* *** **** ******** ** *** ****** physician ** ** allergist *** weekly ******* injections *** woman ** ******** ** ** why ******** ******** ** substances **** *** off her ********* would ********** ******* *** Which ** *** following ********* **** captures *** rationale *** ******* *************** therapy? (Points * ********** exposure to offending allergens ***** *** ********* *** **** cells **** mediate *** allergic **************** ** ********* ** ***** regular quantities ********** the *** antibodies that ******* the allergic **************** ******** stimulates ******* ********** ** *********** mitigating *** ******** ****************** ** allergens simulate ********** ** *** ***** ****** ******** **** ********* with *********** ** * *********** *** has **** ********* **** infectious ************* ***** of *** ********* pathophysiological phenomena is most responsible for his ********* ******* : ***** ************ ***** ***** is lysing **** ** *** ***** ****************** *** ******* **** of *** * ***** *** becoming ************ into the genomes ** ********* *** ******** *** ********** of ***** ***** ****** *** peripheral lymph nodesThe ***** *********** *** mononucleosis ******** *** ********** ** *********** **** promyelocytesQuestion 33 A *********** ****** ******* has ********* to *** ********* ********** because ** several ****** ** intermittently bloody ****** **** has recently become ***** *** woman *** since **** diagnosed **** * gastrointestinal ***** ********* ** ******* ** ************ anti-inflammatory ***** **** she takes *** *** arthritis The health **** **** would ******* **** ***** of *** following ********** ** **** ************** * ***** ***** *** ******** ***** volume due to her ******* ***** ******* will **** *************** ****** *** ** depletion ** **** storesThe ******* **** be ** **** for ************** ******** ** shockShe **** **** delayed ************ release Question 44 ***** ** *** ********* patients ** most likely to ******* **** *************** ** ****** ****** ** ***** ****************** ******* (MHC)-compatible bone marrow? ******* * *** *********** **** **** a ******* ** epilepsy *** low *** ****** ********* ** ********* **** 7-year-old *** whose ***** **** ********* ********* *** and IgG with increased **** ********** *** ***** ***** cells *** ********* ** *********** ** normal * ****** ********** **** who *** * ********* of *** ********** ******** 55 A *********** ************ ****** got * sliver under *** ********** four **** *** *** affected ****** is *** ******** ******* ******* *** warm ** *** ***** ***** of *** ********* ************* ********* ** **** likely ********* ** response to *** infection? ******* * *************** ** ******** neutrophilsHigh *********** ****** ** ******************** ********* ********** ********************** ** ********** ******** 66 ****** samples **** * ******* **** pneumonia contain an ********* agent that *** a peptidoglycan **** **** expresses ********** ********** ******* in ***** *** ** **** ***** ** ******** *** **** *** does *** ***** **** ******* ** crystal ****** **** ********* ** **** ******* ******* : 3)ChlamydialViralMycoplasmalBacterial ******** ** * ***** has **** ********* **** *********** Which ** *** following other ****** ******** is the child ** **** **** ******* * ********************* *** ******** deficienciesSplenomegaly *** *********************** ******** ** * ***** ************ is providing ******** care and education *** * first-time ********* ****** ** ****** ********* who has * ********* ** a sexually *********** ********* ***** ** *** ********* ********** ** *** ********* ****** demonstrates ** ******** ************* ** vertical ******* ************ *** ********** *********** (Points * ************** *** chlamydia **** *** ******** ***** ** ************ **** ****** to ************ know **** ** baby **** **** observation *** HIV ***** *** symptoms ** *** ***** ********* ** **************** **** could ****** ******** ****** ****** *** ******** or ****** *** birth itself”“Prophylactic ************ **** reduce my baby's ****** ** being born **** ** illness”Question ** As **** of his ********** workup * 77-year-old man's ***** practitioner has ******* ***** **** **** ******** ******** ****** *** *** ** **** ********** ** *** ******* ** *** ****** **** and ** ********** **** ******** *** *** role ** **** his ***** ************ to ******* *** significance ** ** *** *** rationale *** testing ** ***** ** *** following ************ by *** nurse ************ is most **************** * ************* ** the activated *** ****** **** ** **** that **** *** blood cells *** *** ** ********* ******************** ** a ****** **** ** **** **** ********** ***** me ******* *** ***** ******* **** **** ******************* ** a ************ complex **** ****** **** *** ***** ***** to **** *** ** *** **** **** you ******* ** **** ****************** ** the form ** **** **** is *********** in your ***** ****** ** *** red ***** cells that need ************* **** A *********** ****** *** **** brought ** *** ******* care ***** practitioner by her ****** *** ** the ****** persistent sore ****** *** ******* ***** ** the following facts revealed ** *** ****** ******* and *********** ***** lead *** ***** ************ to **** *** infectious ************** ******* * 3)The **** has a temperature ** 381°C ********* *** has ******** ***** ******** ***** *** ****** *** **** ************* work ******* ** ********* white ***** **** ********** auscultation ******* ******** ** her ***** lung fields ******************* **** A *********** *** *** ***** * ***** ** * ************* ****** ****** next ** * passenger who *** been ******** *** ******** *** *** *** *** **** ******** viral particles periodically ***** ** *** ********* ********** would **** likely ****** ** *** *********** ** *** ***** * lymphocytes *** ******** ****** system? ******* * ************** of * ******* ******* ** * ******** ************************* of * foreign MHC ******************* of a ******* ******* bound ** * **** *** **************** *********** of a * lymphocyte **** ********** ** ********* cell ***************** **** A *********** ****** *** adheres ** a ***** **** *** **** ********* **** iron-deficiency anemia ***** ** *** ********* ********** of her ********** blood **** ***** be **** ****** to *********** further ************** ******* * *********** **** *********** ****** (MCV)Decreased ********** *** ******************** hypochromic *** cellsDecreased ************** ****** ******** **** A ****** *** are expecting ***** first ***** have **** advised by ******* ** ******** ********** ********* **** ***** ** ***** ** **** * ****** ****** ** **** ***** *** couple **** approached ***** nurse ************ with **** ******* and are seeking ************* ** ******* *** **** ***** are ******** *** **** **** ***** expect ** **** **** harvesting **** *** can *** ***** ************ **** ******* ** *** couple's inquiry? ******* * 3)“Stem ***** can **** correct autoimmune ******** *** some ********** ***************** ***** *** ** **** ** ********** ******* organs ****** *** **** ever *************** ***** can be **** as * ****** ** ******* ***** *** the ****** ***** ********** system”“Stem ***** *** **** ***** some ******* *** ******* but they must come from **** ***** ******* or ****************** 1414 * 23-year-old *** has ******** a ****** ********* ** ************ ********* ** ***** ** acute ********* pain *** ***** practitioner ********* care *** the man ** ********** **** ***** it ** unpleasant *** ************ ** his ******** ** playing a **** ** *** body's ***** against *** underlying infectious ******* ***** ** *** ********* ******** points should *** ***** ************ ********* **** *** ******** for the ******** ******* : ***************** can **** ** ****** the **** ****** ***** **** **** been ******* by *************************** **** ***** your body on *** **** ** ******* *** ******* ****** ** *** **** of ************************ helps your **** ** ******* the ***** ********** ** fight the infection”“Inflammation ********** **** in *********** *** initial ***** of the **** ****** ** **** ******************* **** * *********** ***** ** suspected of having *********** ******** ***** ***** ** the following ******* ** her ********* ***** **** to rule *** ******* lymphoma? (Points * ***** ******** ********** ** ********* ******** ************* ***** ***** ******** *** ******* in a ***** number ** ********* in *** ********* ********* ******** ** ************** ***** has **** ************ ***** complains ** ****** ************ fatigue ******** 1616 ***** ** *** following ********** most ********** ******* ** ****** of ********* ****** ********* (Points * *** and * ********** *********** begins ** *** **** ****** *** ends ** *** peripheral ******** *********** cells *** *********** *** ******** **** *** bone ****** ** ***** ********* ********* ***** ** the ***** ***** initiate and regulate *** process ** ***** **** ******************* bypass ******** *********** *** *** *********** ******* ** the ********* ************** **** * ***** ************ ** explaining ** a *********** male ******* the damage that ************* ************ ***** ** to **** ****** ***** ** *** following ********* would ******** *** ***** practitioner's ************ (Points * ******** *********** results **** neutrophil *********************** ********** ******** ***** ** pulmonary ****** *********** and ********* ********************* *** ****** to digest the ******** ********* ** ****** ********************* are *********** ******* *** ************* tuberculosis antigensQuestion **** * ********** **** *** *** repeated ear *** upper *********** ***** ********** ***** *** *** **** * ************ *** determined a ********* ** transient ********************* ** ******* **** ** the physiological origin of *** ******* ********* infections? ******* * ********** production ** plasma ***** ** *********** because ** impaired ************* between * *** * ******** child *** a ********** ******* ** ************** * (IgG) ********** and her body ** **** slowly ********* to ******* **** **************** ***** was **** **** ************** * (IgA) *** ************** ***** ********** ********** ************ infectionThe ***** ***** *** ******* ********** ***** ******** ** normal B-cell antibody ****************** **** ********* * ****** ** measles a 5-year-old **** ********* scattered ******** **** ******** body ******** *** was ********* **** ****** **************** purpura ***** ** **** ** *** ********** workup ***** **** *** performed ***** ** the ********* ******* is most ****** to ** considered ********** ** the ****** **** team? ******* * *********** ************** levelsDecreased platelet countNormal ******* * ************ ********* levels ******** **** * 60-year-old **** ******* with an ***** ***** ********* ** ********* ********** ******* *** ***** practitioner ** teaching *** ****** ** *** patient ***** *** ******* ******* of *** ********* and *** **** **** it ******* **** ***** anti-infective ********* Which of the ********* teaching ****** ****** *** nurse ************ ******** ******* * *************** *** help **** father's ********** ***** ******** ** his ******** cells ******* ********* proteins **** ***** *** spread ** *** ************************* *** **** limit the replication ** *** ***** ****** affecting **** ********************** helps **** ******** **** recognize ******** ***** **** *************************** *** ******* **** ******** ****** ****** ** *********** ******* ****** cells **** ****** ****************** 2121 A *********** ***** ******** **** ***** *** painful ******* ******** ***** nodes *** ***** **** ********* ************ with * ***** to the **** She most ****** **** ******* * *** mild ********* ********** ****** ********* infectionA **** viral infectionA ****** ****** ********* ******** 2222 A ***** ************ ******* ** ************* herself **** the ********* ********** ** ** acute ******* **** of a university hospital ***** of *** ********* ******** would *** ******* ********* as ***** ***** likely ** have a ********* ** antiphospholipid ******** ** his ** *** medical ******** (Points : 3)A *********** obese **** with left-sided ********** ********* ** * *************** ********* 90-year-old ****** ******** ** a ********* **** facility who *** been experiencing ********* ******** attacksA *********** ****** with * ********* ** left leg *** and * ********* ********* 21-year-old male **** * ********* of cellulitis and ********* endocarditis ********* ** intravenous drug useQuestion **** *** ***** **** ** * *********** male ******* **** a ********* ** ***** ******* ********* to alcohol ***** ********* low ****** ** albumin Which ** *** ********* phenomena ***** * ********* be most ********* in ************* ******* : ********** immune ***************** ****************** thermoregulationFluid ********** Question **** A ***** ************ ** ******** her colleagues ***** the role ** ********* ** * ******* ** *********** ***** ** *** ********* teaching points best captures ** ****** ** the ********* *** ****** ** ********** (Points * ****** ********** ******** *** **** ****** ******* ** ********* ******* * **** **** ****** ***** *********** ***************** production ** ******** **** time but effects *** ***** **** serum ****** cross * particular ******************* cytokines *** ******** ** ******** ********** *** different ***** are ******* of ********* *** **** ********************** ******* are ************* ** that ********** ** *** ********* ********** of other ********* **** ******* ****************** **** Which ** *** ********* ********* ***** be ***** ****** ** ****** ** activation ** *** ********** ******* ******* * 3)Recognition ** ** ******** ***** ** the ******* of * microbeThe binding ** mannose ******** on microbial glycoproteinsActivation ** Toll-like ********* (TLRs) ** complement ************** *********** of ********* ********* *** ***** practitioner for * ********** ******** ** responsible *** providing *********** ******** *** patients *** *** ***** ** undergo * ******** ****** ****** ***** ***** ** *** ********* ******** points **** conveys an ****** ** *** ***** *********** system? (Points : ********* ***** ******** ****** widely ******* ******** *** ***** *** between ********* and ******** circulation”“Only ****** one ******* of your ***** ** ** **** heart ** any ***** *************** ******** *** ***** ****** ******* mimic *** ******* ** *** ***** location ** *** *********** ********************** *** *********** ******* ****** at **** **** of *** ***** **** equal each ***** ** ****** ******** ***** *********************** 22 * ******** ********** ** * *********** ****** patient indicates that *** ***** pressure ** her **** ** ***** **** **** ** her **** *** **** *** ******** ***** ** ****** ** *** **** *** **** ** *** ***** In ******** her femoral ****** are weak *********** Which ** *** ********* ************* ***** *** **** ******** ** **** ****** to suspect? ******* : *************************** hypertensionCoarctation ** *** ******* adrenocortical ******** ******** 33 ** **** ** *** diagnostic ****** *** * **** ******* **** * ******* history ** cardiovascular ******* *** **** **** *** identified *** need for a ****** ** the electrical ******** ** his heart insight into the ********** ** *** ********** *** ******** ************ *** ******* ** *** ***** Which of *** ********* ****** ** ***** ** **** likely to ******* the needed **** *** *** ********* *** care? (Points * **************** *** **** ECGAmbulatory *** ******* *** ******************* ********** levels chest auscultation ********** ********* ******************* *************** ******* CT ******** ****** *************** ** ** ***** ***** female ******* *** ********* with a *** ***** ** ********* ** breath *** *** patient's ***** practitioner *** ordered measurement ** *** BNP ****** along **** other ********** tests **** ** *** **** accurate ********* *** *** nurse ************** ****** of ***** work? ******* : ***** is released ** * ************ ********* ****** ***** ******* and ********* it can help ************* the ********* dyspnea from * respiratory pathologyBNP ** an ******** ********* ** *** ************* of *** *** ****** ** ************ *** ***** failureBNP ****** correlate **** *** ********* **** of ********** ********* deficits ********* ** heart failure *** consequent ***** ********** becomes ******** ** ***** ** ******* asthma Cheyne-Stokes ************ *** acute pulmonary ***** and *********** *** ***** *** ******** ** ********* *************** ** A ******* in the ********* **** **** *** a blood ******** ** ***** and *** **** ******* **** accompanying *** ****** ******* in ***** ** ************* *** patient **** *** arterial and ****** dilation *** * ******** ** ******** vascular ********** What ** **** ********* most ****** ********** ******* : 3)Hypovolemic *********** *************** shockObstructive shock Question 66 * ****** of patients **** ********* ** the ********* ********** ** *** **** ** ***** **** ********** **** *** ************* ********** of myocardial ********** ***** of *** ********* ******** ** ***** ****** ** **** an ********** myocardial infarction ******** (Points * *** *********** ***** *** ** complaining of shortness ** breath *** vague ***** *********** *********** *** *** *** ********* **** fatigue ****** and vomiting and **** moist ***** *********** *** *** **** up with ********** **** **** is ********* ** his **** *** **** *********** man *** has ***** **** ***** and ***** **** **** ** ************ **** ** moves *** relieved **** ** restQuestion 77 * *********** *** **** * ************* ********* ** ********* hypertension ** meeting with *** ***** practitioner *** ********* ***** ************ ***** anticipate that ***** ** *** ********* ********* is most likely ********** ******* * 3) *** patient's *************** ***** *** releasing *********** ** * ****** ** *********** stimulationEpinephrine **** his ******* gland ** initiating *** renin-angiotensin-aldosterone ***************** is ******** ** effect ** *** ************** *** ************* resulting ** ******************* ********** of *********** I to angiotensin ** ** *** lungs ****** ********* ** blood ******** *** ****** reabsorptionQuestion 88 * 66-year-old obese *** **** * ********* ** ******** heart disease *** been ********* **** heart ******* that *** **** **** *** characterized ** attributable to ******** *********** ***** ** *** ********* ********** ******** ** ************ **** his ********** (Points * ***** ******* ***** ******** ** normally ** *** range ** 150/90 *** ** ************** ********* *** ******** fraction is ****** ************* ****** is ****** **** ****** *** *** ******* heart **** ** ******* and ** ***** *** ******** ** ********* volume ********** following ******* **** of *********** ***** ********************** ******** *** **** ******* *** ************* ***** **** normalQuestion ** Which ** the following assessment findings in * ***** admitted *********** male ******* would be most ****** to ***** his nurse ************ ** suspect polyarteritis ******* ******* * ***** ***** blood **** indicates polycythemia ********* *** ***** ******* *** ************ (elevated ***** ********* ***** blood pressure ** 178/102 and ** *** ******** ***** function ******** man ** ******* short ** breath and *** ****** ********** ** 87%The man's *********** ** ******* and ** ** diaphoretic (heavily ***************** 1010 * 6-year-old *** *** **** ******* ** the ********* ********** by ********* ***** *** mother discovered that *** heart rate *** ***** **** * ******** even ***** ***** The ***** *** ********** ** be ** ****** ******* *** his ****** is seeking an *********** **** *** health **** **** ***** ** *** ********* ****** ****** ******** an *********** to *** mother? ******* * 3)The child is ************ * ******* ****** ** *** ***** ********* resolution ** *** ******* ** dependent ** spontaneous recovery and ** resistant to pacing **************** ***** ** ****** to **** * ****** *** ***** **** the rapid ***** ******* ***** ***** are ************ abnormal sympathetic ******************* **** * ******* *** ******** ****** ** his *********** following * ***** vehicle accident Which ** *** following ************ ****** the ***** ************ ** **** ****** ** **** **** ******* : 3)Impaired physical ********* of *** **** ventriculeIncreased ******** during *** ********************** ************ ********** from ********** ***************** ********** ** myocardial contraction ******** **** Which ** the following situations ******* ** *** ********** **** ***** ** ********* *********** would be **** ****** to *********** ******* ************* ******* : ********** ** the ********* *********** *** *** ***** side of *** ******** ***** fall **************** oxygen tension ********* causing ******** of ********* **************** of the ***** arteriesSystemic ******** ********** and left *********** pressure *** **** ******************* ******** resistance ******* ** ****** ********** in the pulmonary ******** rises **** *** ****** ** the infant's ***** ************ 1313 * *********** **** ******* presents ** *** ********* department complaining of **** ** *** **** that ** *********** **** ** ***** *** ***** *** ********* pulses *** ******* ******** *** his leg distal ** *** **** ** ********** ******** What ***** *** care ********** *********** diagnosis *** anticipated treatment most likely *** (Points * ******* ******** ********* that **** ** ******* **** ****************** disease **** **** require ************ ************************** ********* ******* ************* ************ ************ **** ******** ********* that **** ** ******* **** *********************** **** * patient ** ************ ******** circulation ********* ** ********* ******** ******** pressure ***** ** *** following ********** ** the **** ******** *********** (Points * 3)Increased ******* due ** ******** ************** ********* because ** ************ ******* the **** ***************** ************* *** ** ****** resistanceSystolic ********** ******* of ******** ******** Question **** A ***** ************ is ********* care *** ******* ******** ** * ******* **** ** * hospital ** ***** of *** ********* ******* situations would *** ***** ************ be most ****** to **** *** hypertension as * contributing factor? ******* * 3)A *********** *** *** *** * heart valve ********* *** ********* ******* *********** woman *** has had an ******** ****** and has ********** one-sided ********* 44-year-old *** ******** * kidney ********** *** requires ************ ***** ***** *** weekA *********** ***** **** ****** ********** ****** *** ********** limited activity ***************** **** * ******** normotensive ***** ******** *** the ***** **** develops ************ *** headaches ** ** ****** ********* *** blood ******** ** ******* mm ** *** she *** *********** **** ***** **** should be ordered for **** ******* : ******** angiotensin * *** ** *** reninUrinary ****** *** potassiumPlatelet ***** serum ********** and ***** ************** ************** and *********** Question 1717 * nurse practitioner is *********** a group of older ****** ***** *** risks ********** **** **** *********** ***** ** *** following ******** ****** ****** the participants *** ** ********* **** ***** ********* ***** the ******** ******** ******* * 3)“Remember the *** ** *** and *** *** ** *** correspond to **** ****** and *** ****** ** **** health”“Having **** *********** ********* **** **** ** ********** diabetes *** ********* ***** ***************** and being overweight ********* your **** ** ******* ****************************** family ******* ** ******************** is ********* *** ***** *** ****** *** can ** ** ********** for * **** ********* *************** 1818 ** ******* ** ***** ********* ** * *********** ****** *** died ************ ** ***** ******* ***** ** *** ********* ********** ** the ************* ****** is **** suggestive ** * possible ******* of ****** controlled blood ********* ******* : 3)“Scarring ** ******* suggestive ** ********* ******* ***** ********** ** ********************** ***** hypertrophy ***************** wall ******* ********** ** ****** stasis *** ********************* sclerosis ** subcortical ***** ******* ******** ******** **** * ***** ************ has ******* *** measurement of * ******* ********* electrolyte ****** ** **** ** *** patient's ******* blood **** Which of *** ********* ********** best ******** *** ********** of ********* ** the normal ********** ******** of the patient's ****** ******* * 3)Potassium ********* *** ********** of *** ********* the ******** **** ******* ** electrical ******************** is ******* ** ************ *** *********** the ******* ******** ********* ** ******* muscle ******** impermeability ** cardiac cell ********* ** ********* ****** for ****** ********** achieved by the **** ** sodium ******* ********** ******** ** *** ********* *** for *** ****** *** ****** the cell ******** ******* in *** ********** ** ** ****** ***************** **** ** ***** ** *** ********* patient situations would * ***** practitioner ** most ********* ** preliminarily ruling out ************ ** * ************ ********* to *** ********* ****** ********* ******* * 3)A *********** man ***** *** was ************* ** ********** * wave ********** ** ********* but ***** * waves **** ******** normalizedA 77-year-old with ********** ** *** S4 ****** ** ********* ***** **** *** * ******* of ****** fibrillationA *********** ***** *** who ** *********** ** chest **** **** ** *********** ** deep *********** *** ** radiating to *** **** *** ******** ridgeA *********** ***** ***** ********* ***** work ********* ******** white cells erythrocyte ************* **** *** ********** ******* ************** **** ****** * ******* ******** *********** ** * 66-year-old ***** her nurse ************ ***** * ********* ********* **** *** ****** *** woman for ******* ********* *** ***** ************ ** ********** *** ********* ** *** ******* *** is unfamiliar **** aneurysms ***** ** the ********* ******* ** *** pathophysiology ** ********* would ******** *** *********** *** ***** provides? ******* * 3)Aneurysms are ******** * ****** ** ****** controlled diabetes ******************** ** * ******** ********** contributor ** ******************** **** ** ******** *** ******** asymptomatic ***** *** aneurysm rupturesAneurysms *** ******** ** resolved **** lifestyle *** **** ********************* **** A *********** patient's ************** ******* a ************* **** ********* ****** chamber ****** *** a normal ******** ******** **** *** ***** What ** **** patient's **** ****** diagnosis? ******* * ******** ***** ******************* ***** ************** ***** ************** ***** regurgitation ******** 2323 A ***** practitioner ** teaching a ******* ** about *** physiologic ***** *** ****** ** *** *********** *** neurological ******* that *** ********* *********** ***** ** the ********* ********* by *** student ***** ******* correction by *** ***** ************* ******* * ******* ****** **** ********* increases tension decreases”“Smaller ***** vessels require **** ******** ** ******** **** **************** ******* *** ****** ****** *** ******* the ******** ****** ** **** it ***************** *** ****** ********* ******** ****************** ******** **** An *********** ****** ******* of * ********* **** ******** *** * ******* ** congestive ***** ******* *** ***** ************ caring for *** patient *** ********** her ******* up at ** angle ** *** *** ** ********* her ******* venous distention *** ** ******* ****** ********** a useful indicator for the ********** ** *** patient's condition? ******* * *********** ******* ****** causes engorgement ** ******** ***** ******* of ***** *** ******* **** ** *** ** *** largestBlood ***** ** into *** ******* vein because there *** ** ****** ** the point of ***** into the *************** ******** ** ********** **** ********* stroke ****** *** ******** ************* ****** *** not ******* of preventing backflow ** ***** ** ****** ****************** 2525 * *********** **** ** ************ *********** shock ********* a fight ** ***** *** ******* artery *** *** **** a ****** ****** **** immediate ********** are likely ** most benefit the **** (Points * ************ ** compensatory pulmonary ***** *** heart arrhythmiasInfusion ** ************ ** ****** perfusion *** ********* ** improve ***** ********************* of ****** ****** ** ****** ******* ** ******** *** ******** ******************* ** ****** *** *********** ** ******* ********** ***** ** *** ********* ******** who ********* ** a ******* ******* ****** ** **** ****** to ** ********* **** * ************** ****** **** a common ***** (Points * *** *** complaining of general fatigue * headache *** ****** pain **** a *********** of ******** woman ********** with ******* lethargy and copious nasal *********** *** **** * dry stuffy nasopharynx * sore ****** and *********** of 989°FA woman *********** ** *********** ***** *** *** * ****** ***** and ******** painful ***** airwaysQuestion 22 * *********** ***** developed calf **** ****** a ************* ****** ******** ** acute ********* ** ****** upon ******* ** her *********** She *** ************ ********* with * ********* ******** **** ***** resolved **** ************* therapy ***** ** these ********** **** characterizes *** underlying ******* ** *** PE? (Points : ***** *** ***** ** breath because *********** *** occurring *** ********* *** ************* *********** ** normal ********* *** compromised ventilation ****** ********** ********* * ********* ******** ***** resulting ** impaired oxygenationImpaired *** ********* ****** ******** membranes resulted ** ******* *** hypoxiaQuestion ** * ********* ****** ** ********** *********** *** developed **************** ********* (BPD) *** ** showing ***** and symptoms ** ********* *** **** compliance *** respiratory ******** ******** (RDS) ***** ** *** ********* ** *** **** ****** *********** to *** ******** ******* ****** ******** ******* * 3)High inspired oxygen ************* *** injury from ***************** ****************** ** ********** *************** ** the infant in uteroInsufficient ********** ********** *** ************ ********** ******************* ************ ****** ******* Question ** Which ** *** ********* ********* ** * ********* **** facility ** **** ****** ** be ********** the ***** *** ******** ** ******* obstructive ********* ******* ******* ******* * *** *********** lifetime ****** *** ** *********** of ********* of breath *** pain ** **** inspirationAn *********** ****** *** *** increased ******** *********** a ***** *** ********* white ***** ******* *********** **** who *** * ********** ***** and ********* respiratory ************ 88-year-old ****** who *********** ***** shortness ** ****** and ****** ************ **** exposed ** tobacco smokeQuestion ** * 66-year-old **** ******** to *** emergency ********** *********** by *** **** who claims that he has **** ****** ******** *** *** is *********** of a ****** ***** of severe weakness *** ******* *** *** * dry ***** *** ******** *** *********** ** 1028°F *** his ***** **** ********* *** sodium ***** ** *** ***** ******* ** 135 to *** mEq/L) What **** ** *** ********* ***** ************** most likely ********** ******* * **************************** ******************** diseasePneumococcal pneumonia ******** ** * 51-year-old female ******* *** ** * days postoperative on * ******** **** ** * ******** is at risk *** developing *********** as a result ** ***** ******* immobile Which ** *** following ******** points ** her ***** practitioner ** **** ************ ******* : ********** in *** ********* *** **** of ***** ************ ******* **** ***** and ***** ****** so **** ********* *** *** ** change ********* often”“You ****** ******* ****** and ***** ** **** your ***** expand ** **** ** ******** ***** ****** ** bed”“Make sure **** *** stay hydrated and walk ** soon ** possible to ***** *** ****** to ****** * ***** ************** prescribe ************** *********** **** will help open ** your ******* *** ***** **** oxygen in”Question ** Due ** ************* * **** ************* ******* *** **** ****** ** ******** *** several days following surgery *** has developed *********** ***** of *** ********* ********* ***** *** **** team ********** **** relation ** his health ******** ******* * ************** ** *** ******** ******* ** *** ******** region ** *** ************* ******** for *** left **** ** *** ********* ************** blood **** to *** **** ** atelectasisRedirection of ***** **** **** **** *** **** regions **** *** hypoxicQuestion ** ***** ** *** ********* ******** findings ***** ** **** ******* associated with * ******* *** *** interstitial **** disease rather **** COPD? ******* : 3)Audible ******** ** ******************** ********** **** ************** *********** rate with ********* tidal ************ ********** of alveolar tissue ******** ** * **** ******** smoker has **** ** * ****** ** ******* *********** pulmonary ******* ***** ** the ********* phenomena regarding *** ******* ***** *** care **** have **** ********** ******** ** the ***** ***** ** *** ****** ******* : *************** ** ******* killer (NK) ***** in *** alveolar ********** ******* ** alveolar *********** ** septal ********** ********* presence ** ********* ** the ************** spacesCompensatory regeneration ** **** I ******** cells ******** **** * *********** male ******* *** suffered * **** injury ****** * ***** on his motorcycle and * deficit that *********** **** ******** ** an ******** swallowing mechanism ** has **** ********* ********** ********* ***** ** *** ********* statements most ********** ******* ** ****** ** *** condition? ******* * 3)His ********** is ************* epiglottis ** ******** *** larynxHis vocal folds **** **** compromisedHis tracheobronchial **** ** intermittently ********** ******** 1111 * short ********** *********** *** ******** to *** emergency department with left-sided ***** **** and * cough ** **** **** *** pain ******* ******** just after lunch and **** ********* and coughing make it ***** He ****** ****** ****** He ** ********* ********* and ******* *** ********* **** **** ** *** ** having * ***** ****** Breath ****** *** ****** *** ** ** not ******** Which ********* ** **** likely causing *** symptoms? (Points : ************ ********************* pneumothoraxPleuritis ******* ** infectionObstructive *********** ******** 1212 Which ** *** following statements best ******* ** ****** of the respiratory ********* **** govern ventilation? (Points : ************** ******** slightly ******* **** ** *** ******** ******* ***** **** ****** ******** pressure ** *** lungs **** *********** ** ************* ***** *** ********* from ********** ** ******** ******** ************** **************** intrapleural ******** holds *** lungs ******* the ***** ************ 1313 *** ****** ** * ********** boy *** has ******** **** ********* **** childhood ****** has **** ** *** ********* ****** ** learn more ***** *** ***** ********* ***** of *** following ******** ****** ** **** ************ ******* * ************* has ***** that ******* may ******** ** * ****** ** **** children's *************** **** ******** ********* that **** ***** ** having ** asthma attack ** ******* ********************** that **** child *** ****** will ****** be *** ***** **** ** defense“Your son will likely **** to ***** ** avoid ******** and ***************** **** ***** of *** ********* ********* ** **** likely ********* ****** * ******* ******** ***** of **** ************ (Points * ********** alveolar **** are ********** and ********** ********** is ********** single-capillary ******* exists *** *** ***** *** capable ** ************** conducting ******* *** formed *** *********** ** *** *** ***************** alveoli *** ****** *** *** bronchi *** *********** ****** **** ************** **** A 71-year-old ***** ** ********* ** ****** ******* *** *************** *** ** *** diagnosis ** emphysema ***** ** *** following ********* ***** *** **** **** ** **** justified ** ****** out? ******* * *********** elastic ****** due ** alveolar *************** ******** lung ****** due ** impaired ******** ******************** ******** **** ***** *** ** ******* tidal volumeIncreased ******** **** ***** *** ** ********* intrapleural pressureQuestion 1616 A ********** **** for ****** ******** ** ******* * ***** sweat ***** ****** ******** ***** *** ***** ****** **** *** ********** ************* Question **** * ******* *** ********* **** ********* of breath and ********** ******** ****** *** **** ********* **** ********* ******** * disease ************* by ******** ** the ******* **** ***** her **** team anticipate **** ********* her breathing? (Points : 3)Rapid deep breathsWheezing breathsShort shallow ***************** ******* Question 1818 ***** of the ********* ******** are *** ********** ***** **** factors *** *** *********** ** ********* ******* ******* * *** patient *** is immobilized ********* ********** ******** ******* *** *** ******** Cl– and Na+ regulationA ******* *** ** ****** amiodarone *** *** ********* ** * cardiac arrhythmiaA patient *** is * ****** *** takes oral contraceptivesA ******* *** ** ********** radiation ******* *** *** ********* ** ****** cancerQuestion **** ********* a ****** ***** ****** * ******* *** *** **** ***** * **** ******** generator ***** ** experience dizziness *** ********* *** *** ********* **** ****** monoxide ********* **** is *** **** ** hyperbaric oxygen treatment for ****** monoxide poisoning? ******* * **** increase the ****** ** ****** dissolved ** ******** ******** *** ********** of ******* ************ ********* the release of oxygen ** the ************* ****** ***** ** **** hemoglobin ******** 2020 * ***** ************ ** ********** patient ******** ***** *** influenza virus **** **** patient *** has **** to *** ****** ** ******* **** ****** ******* ***** of *** ********* statements ** patients **** ******** an accurate understanding ** *** *** ****** (Points * 3)“I ***** come down with ***** ** ********* ********* ** * ****** ** * *** *** ********** **** ** *********** ** especially important ***** ***** ****** *** ***** **** *** ***** the *** once * get sick **** it”“The ******** ** ******** ** ******* **** and ******** lots of fluids ** outdated *** actually ********************* *** ******** ** ** ***** ** ******** ** * ********** gets immunized ******* *** ************** **** A 77-year-old ******** ****** has **** diagnosed **** a ***** ** *** **** ** *** **** of ** *** tubercle ******** site ******* ** * peripheral **** ** *** *********** ****** **** ** **** patient's **** ****** ********** (Points * ********** cell carcinomaSmall cell **** cancerLarge **** carcinomaAdenocarcinoma ******** **** ** * ****** ** *********** a ********* ********** ***** *** producing ************ ******* ** ***** ************ *** ********* mucociliary ******* ** compromised ***** of the ********* changes ***** * **** ******** **** ********** ********** as a ****** ****** of **** change? ******* : ********** function ** the ********* ************** levels ** ****** saturationIncreased ******* ** bacteria in the ************** carbon ******* ****** Question **** ***** ** *** ********* ********** **** ******** the ******** ** *** ***** ******** ***** of ********* ******** ******* ******* : ***** ********** ** heightened ** * *********** ** ******** to ** ************** ********** ******* ** ******* ********************* ****** and ***** ***** ***** **** ******* ** *********** **************** ********* are released **** ************* **** cellsQuestion **** * *********** ****** ****** ** ********** ** *** ****** ********* ** small **** **** ****** (SCLC) How *** *** ***** ************ **** ************* respond ** **** (Points : ******** **** **** ** very hard news ** hear *** ** ***** **** **** aggressive ********* **** ******* of beating **** *** ***** good”“This ** **** ********* ** **** *** sure *** we have ** ******* to *** ** ** ******* ******* **** ***** **************** **** ***** ** **** to **** *** this news; *** **** ** talk ** you ***** surgical options however”“This ** a ********* ********* ** ******* *** there ** * ****** **** *** ****** may ** **** ******************** **** ***** * ***** ***** ****** ** *** chicken ***** a *********** ******* ****** *** developed * ***** ****** and vomiting ***** ****** out **** common ****** ******** his **** ******** ********** makes a ********* ** ************** ***** of the ********* ********* is **** likely ****** ****** ******* * 3)Toxin production ** Histoplasma ********** is triggering an ****** **************** production ******* *** ********* ***** is ******* ** *** ********* *** and the ********* of the ************* ********** initiates ** ********** response that ******** *** ********** symptomsMacrophages *** **** to ****** *** ********* ***** **** *** *********** *** ***** ******* ***** * *********** *** *** **** ********* **** ***** ******* ***** ******** episodes ** ************ ***** **** ** ****** weeks The *** states **** don't **** *** **** could happen ** ** ***** *** ** careful ***** ****** * healthy ******* **** is the most *********** response ** *** ***** ********** ******* : 3)“Your **** *** have played a part ** this *** ** fact ******** is ****** ********* ** *************** you *** *** ********* your **** ** stone ********* *** **** ***** ******* **** ******** *** **** ********** are ***************** ****** ***** need to ****** **** **** *** *** **** *** **** stones in *** ****** ****** ** **** **** risk ** ******* **** ** *** ***** **************** diet ***** ** ******** healthy *** **** ****** of normally ********** minerals **** ******* and ********* can lead ** ***************** 22 ** 87-year-old **** ******** of ** ******** ****** facility *** **** ************ ********* of urine ***** *** **** ******* ***** Which ** the following ******* ** the **** ********* ** *** ******** ** *** **** ****** priority? ******* : ** Performing * ******** *********** and history ** ********* the exact cause *** ********* ** *** incontinenceProviding ******* ********* ******** ** *** **** **** occasional incontinence ** * ****** age-related changeTeaching the ******** about ********** **** ********** ******* *** *********** **** *** be **************** *** ******** *** ******* ********* *** exercises ** improve ******* ********* *** ****** floor toneQuestion ** * 34-year-old **** patient has ********* ** ***** ******* ascites and hepatic ************** ********* to alcohol ***** The ********* ****** ** *********** the **** **** ***** why *** ******* is ** ***** **** ****** he is ************** *** ********* ***** ** the ********* ********** **** ********** underlies *** *********** **** * ****** of *** **** team ***** ******* to *** family? ******* : **** ********** ****** ** ************ fluid is ************ ** *** patient's ********** ************* ********* of the ************* fluid compartment ******** far **** ***** **** normalThe ******** ***** ************* ***** compartment ** ***** ***** ** becoming ************************* ****** is accumulating ** *** patient's ********** ************** 44 A ******* is ******* ** *** emergency department **** complaints ** ********* ** ****** Assessment ******* * **** ******** ***** severe ***** and audible ******** in *** ***** **** ****** *********** What ** *** ********* **** ****** ********** (Points : ******************* volume excessHypocalcemiaHyperkalemia ******** 55 Following ******* **** in an ******** ***** * ******** ******* has ******** to *** desired ** Which of *** following ********* ***** **** *********** ** *** ********** ** the patient's health *************** : ********** of *** *** ** ************* renal ********* *** ************ ** ************ and ******* buffer systems ** the ***** **************** ** HCO3– ** *** ******* ******** 66 Which ** *** ********* ******** would be considered ** **** * *********** risk of ********** the prerenal **** ** ***** ***** failure? ******* * *** 22-year-old male *** *** **** * ***** amount ** blood following * ********* ******* 41-year-old ****** *** ** admitted *** *********** antibiotic treatment of pyelonephritisA 79-year-old male with diagnoses of poorly ********** diabetes ******** and ********** ***** ******** 20-year-old male *** ** admitted *** ********* ** ** ******** ** * *********** drugA *********** male **** * ********* ** benign ********* *********** ******* *********** female *** *** **** ******** for ********* ** dehydration ************ *** ******************** ** An *********** ****** has ************* hypocalcemia secondary to ****** ******* *** **** shortly be ****** **** an ******** ****** ******** from *** own ********* ***** ** *** ********* ******** should ***** at *** ******** ** ********** ** ******* **** (Points : ****** of appetite and ********** ** ************** ****** *** ********** ** ********** fluid intake *** ***** ************** *** ************** ** * *********** *** *** made ** *********** ** *** his nurse ************ ******* ** *** **** awakening ***** ** **** ***** ******* to void *** ***** *** * ****** **** to **** **** little warning ****** the day **** ** *** man's **** ****** ********* *** ******** ********** **************** ******** ******* * ******** incontinence *** to ****** ** *** ********** ****************** ******* that *** ****** from **** neurogenic *** ******** ***************** ******* due to ************ ******** ********* ******** **************** ************ **** can ****** **** ************ ** *** ***** ******* *** ******* and *** ********* ******** *************** 99 A 68-year-old ***** with * *** ***** ** ******** ******** has ******** begun ********* ***** ***** ** *** following physiological phenomena ***** her **** ********* **** realistically ****** ** be ********* ********* ** a ****** ** *** ******* retention? ******* * ************* ** *** ******* ****** and ********* ******* wall ****************** ***** production *** *********** ****** ********* by *** ********************* ******* stretching and **** ******** ***** volumeOverflow ************ *** **** ** *********** ************* 1010 ***** ** *** following *********** ** ** the highest risk *** ********** * urinary ***** ********* (UTI)? (Points * 3)A *********** *** **** * ******* ** ************** ******* who is ********** ** the ******** **** a ******** artery ****** graftA 66-year-old *** ********** ******** *** *** ********* ** ******* ***** failure ********* ** ************* *********** *** **** **** ***** output *** ** ************ ******* ************** 30-year-old ***** ***** **** poorly ********** ******** mellitusQuestion **** * ***** educator ** *********** *** ****** ** * renal unit ** * ******** Which ** the ********* ******** ****** should *** ***** ******* ** part ** a ****** of ****** ********** function? ******* * ************* *** ******** structures **** ****** ****** the high ******** that ****** in ********* **** ********* ** *** ******************** ******** ** very ******* ** *********** ** ***** ****** ***** ********* ** circulation”“Dilation ** *** ******** arteriole allows **** ***** into *** ******* and ********* *** ********** ********** ************* ********** ** ******* ******* an ********* *** a ****** **** **** ******** ** ******** ***** *************** 1212 * *********** ***** ********* with ******** ********* **** **** ****** prior ** ********** **** she *** *** several recent ******** ** urinary ************ *** *** asked her ***** ************ *** **** ** *** case ***** of *** ********* ********** **** ******** *** ***** **** ***** ******** the ******* ******** to *** ******** ******* * ************ diseases like ** often ****** ** ******* bladder ************** *** be unable ** ***** *** ******* filling as * result ** *** ********* ** *** ***** ******* or extrapyramidal ***** ********** with ** ******* detrusor ********************* reductions ** ******* ****** brought ** ** ** necessitate ******** micturitionQuestion **** * ******* **** ****** ********** diabetes ******** ******** ** *** emergency ********** **** suspected ************ Which ** the ********* ********** ******* would ** **** ****** ** ******* **** ********** (Points * 3)Low ** ****** increased anion *** **** excessHigh ******* levels decreased anion *** **** potassiumIncreased CO2 increased anion *** **** **************** CO2 decreased ***** gapQuestion 1414 A ********* **** **** ** ********* **** * ***** ******* ******* a ******* ******** ** ** ********* ****** **** a ********* ********** renal ******** ***** of *** ********* possibilities ** *** **** **** most ****** ** ** able to rule out early? ******* : ***** ** the ******** ******* *** have ****** ** ******* to * ****** ******* kidneys may ** ********* and **** ******** upper or lower poles of the *** kidneys may ** fusedRenal **** ********* *** ** *************** **** A ********** boy who has been **** since ***** *** *** ********* cataracts *** **** ******* ** *** ********* ********** ** *** ****** ******* she noticed blood ** *** ****** ***** he **** voided ********** confirms heavy *********** ********* ** **** as *********** **** **** *** **** ****** ******* ************ ********* most likely *** ******* * ******** **************** lupus ************* glomerulonephritisHenoch-Schonlein ******* *********************** * ******************* 1616 ***** ** the following *********** ***** ** ********** to ** ** risk *** the development of edema?(Points * **** *********** man with *********** ***** ******* and *************** 60-year-old obese ****** **** * ********* of ****** controlled ******** mellitusA *********** ********** ****** *** *** ******** ********* burns ** * *********** ********* *********** ***** *** has ** ****** **************** bleed and ********** ******* *********** ****** **** *************** ********* to ************ *** anorexia *************** **** * *********** ****** **** * ******* of intermittent ***** **** ******** UTIs *** hematuria *** **** diagnosed **** ********* ******** ********** ****** ******* ******* ***** ** the ********* ********* *** **** ****** *********** ** the *********** ** *** ****** ******** (Points : 3)UTIs ******* **** ** ******** ****** response have caused *** ******** *** inherited * ******** *** epithelial **** ** *** ******* ** *********** ********************* hypertension *** ****** ************ are ****** ************* has ********* undersized ******* **** *** ***** ** ******* formationQuestion 1818 * *********** ******* **** a ********* ** end-stage ***** ******* *** **** ******** ** *** ********* care **** of * ******** Arterial blood *** ******** ********* **** the man *** ** ********* imbalance ***** ** *** ********* ********** is most ****** ** ****** in ** ************* *** ******* * ************** or ********* of *** HCO3– ** the ********** ******* *** ****** ******** levelsTranscompartmental ******** ** ** and ********* ionsRenal ********* of ******* ** the ******** ** ****** baseQuestion **** ***** ** the following data ***** * ********* ******** ** ** **** ********** ** ***** renal ******** ******* * 3)Alterations ** ***** pH ********** ************** *********** waste levels decreased ********** ********** **** ************** serum ********** *** ***** **** ******** ***** decreased potassium *** calcium levelsDecreased urine ****** hematuria ********* *********** **** * *********** ***** *** *** had ** ***** *********** infection *** ******* ***** has ********* ** *** ***** ************ **** ********** ** * ****** ***** ** urinary retention *** ******* to *** nurse ************ that *** *** been ****** **************** **** *********** at higher **** *** ********* dose for the **** *** weeks ***** of the following phenomena **** *** nurse ************ **** ****** ******* ** ************ ** *** urinary ********** ******* * ************* ******* ** the **** ******** *** ********** ******** *** ******** ********* contractionAntihistamine ******* ******* ************* ******* *** pons *** *** ************* cordThe *************** effects of *** ***

Click here to download attached files: MN551 Unit 5 Mid Term Latest 2017 April.docx
or Buy custom answer
LEARN MORE EFFECTIVELY AND GET BETTER GRADES!
Ask a Question